About this Issue

Classical liberalism has a rich and complex history. At Cato Unbound, we hope its future will be likewise. This month’s issue takes stock of the past while proposing to examine the way forward, too: What’s next in classical liberal political thought?

Matt Zwolinski and John Tomasi propose a straightforward but controversial approach: Market liberals, they say, can beat the advocates of social justice at their own game. If we recognize a just social system by the material rewards it extends to the least well-off among the working class, then we should recognize that the winner is… free-market capitalism. And the defenders of the free market ought not to be shy about it.

Are they right? If so, how do we make the case to a skeptical public? Joining us are two eminent scholars of the classical liberal tradition, Roderick Long and David Friedman, and one rising new star, Alexander McCobin, now a graduate student in political philosophy and the Executive Director of Students For Liberty.

 

Lead Essay

A Bleeding Heart History of Libertarianism

Readers of Cato Unbound can probably recite the traditional libertarian philosophical position by heart. That position begins with the axiom that individuals own themselves and their labor. From there, it deduces the right of individuals to acquire property in external resources like land and minerals. These property rights are nearly absolute in their moral weight. Indeed, for the traditional libertarian, respecting property rights is simply all there is to justice.[1] Worries about free speech, discrimination, or the environment are, to the extent that they are legitimate concerns at all, ultimately just issues about property rights. Other purported moral issues—such as questions of social justice—are merely the product of a conceptual confusion.[2] Justice is about respecting individual property rights. To the extent that respect for property leaves some individuals poor and destitute, individuals might be called by a sense of charity and beneficence to respond. But the moral justification of free market institutions is logically independent from any claims about the effects of those institutions on the material holdings of the poor.

This view, or something like it, is often attributed to the giants of twentieth-century libertarianism such as Ludwig von Mises, Ayn Rand, and Murray Rothbard.[3] To be sure, there are other libertarians who do not fit this mold so well. Neither Friedrich Hayek nor Milton Friedman, for example, treats property rights as moral absolutes, and both allow some significant role for the state in the provision of a social safety net. But this, many think, just goes to show that these individuals were not fully libertarian, however great and important their insights into free markets and spontaneous order may have been. Mises, Rand, and Rothbard, it is assumed, are the benchmarks for ideological purity.[4]

By this standard, much contemporary academic libertarian thought seems hardly libertarian at all. “Neoclassical liberals” such as David Schmidtz, Gerald Gaus, Jason Brennan, Charles Griswold, and Jacob Levy reject the claim that justice is reducible to property rights, reject the idea that property rights are a kind of moral absolute, and embrace ideas that are anathema to the traditional libertarian view, such as positive liberty and, even worse, social justice.[5] If Hayek and Friedman represent weak or imperfect versions of traditional libertarian principles, neoclassical liberals seem to positively betray those principles.

There is a productive debate to be had regarding which of these views is more defensible on philosophic and other grounds. Our views on this question are no secret.[6] But this is not the debate we wish to enter into here. Our thesis here, and in a book we are currently writing together, is historical rather than justificatory in nature.[7] We do not argue that traditional libertarianism is wrong. Instead we claim that, from a broader historical perspective, traditional libertarianism is really not so traditional after all. Neoclassical liberalism, as novel (heretical?) as it may seem to some, has a better claim to that title. The postwar libertarianism of Mises, Rand, and Rothbard crystallized many of the insights of the libertarian intellectual tradition into a coherent body of doctrine for perhaps the first time. But in that process of crystallization not only impurities, but genuine insights were expunged. The result was a philosophic system that brilliantly exhibited some of the key elements of libertarian thought, but that simultaneously severed those elements from their original historical and economic context, and therefore presented them in a way that would have been largely unrecognizable to their intellectual forbears.

In the remainder of this essay, we will discuss one particular way that neoclassical liberalism has a better grounding in the libertarian intellectual tradition than the libertarianism of Mises, Rand, and Rothbard. It is not the only contrast, but one of the clearest and most important differences between these two schools of libertarian thought has to do with the proper nature of concern for, and obligation to, the working poor. On this issue, the neoclassical liberal position is that the fate of the class who labor at the lowest end of the pay scale under capitalism is an essential element in the moral justification of that system. And this position, we will argue, has a far more solid grounding in the libertarian intellectual tradition than the justificatory indifference to which the postwar libertarians are committed. Libertarians have long linked the fate of the working poor under capitalism to the justice of capitalism itself. Sometimes this has been done out of a kind of religious conviction; sometimes as part of a pluralistic moral philosophy that takes considerations of need and sufficiency to have significant moral weight; and sometimes out of a more general commitment to institutions that serve the interests of all persons. This last approach, in particular, has been advanced in recent times by neoclassical liberals committed to the principle of public justification, a principle with deep roots in the libertarian intellectual tradition.[8]

John Locke (1632–1704) is today widely regarded as the father of natural rights libertarianism, a direct inspiration to the likes of Rand and Rothbard. But Locke’s work stresses both the idea of public justification and concern for the poor in a way that is largely absent in these later figures. A fundamental premise of Locke’s political thought was the essential freedom and equality of human beings. There is no natural authority or subordination among men. As a result, any authority or subordination that emerges—including the kinds involved in enforcing a system of property rights—must be justifiable to each man by his own lights. Thus while Locke famously defended the right of private property, that defense was contingent upon the satisfaction of the so-called “Lockean Proviso” which required that appropriators leave “enough and as good” for others.[9] The point of this proviso, in Locke’s own words, was to ensure that property was not appropriated in a way that caused “any prejudice to any other man.” For Locke, it was an essential element in the justification of a system of private property that even the poorest would do well in it. Thus, “a day labourer in England,” Locke tells us, “feeds, lodges and is clad” better than a king in America—that is, a (naturally bountiful) place where this system is not in place.[10]

Adam Smith’s (1723–1790) concern for the laboring poor was central to his attack on mercantilism, a system that Smith thought exploited the poor and prevented social mobility. Arguing for his system of natural liberty, Smith defines the “opulence” of a nation in terms of its ability to provide real opportunities for citizens of every class. “It is but equity, besides, that those who feed, clothe and lodge the whole body of the people, should have such a share of the produce of their own labour as to be themselves tolerably well fed, clothed and lodged.”[11] Indeed, Smith was so concerned to demonstrate the positive material effects of commercial society upon the poor that contemporaries such as Robert Malthus criticized him for insufficiently differentiating between the wealth of nations and “the health and happiness of the lower orders of society.” In other words, Smith’s problem was that he cared too much about the poor.

If classical liberal luminaries such as Locke and Smith defended market society by referring to the material benefits of this system to the poor, why does this tradition still get such a bad rap for neglecting the less fortunate? If we are looking for someone to blame, there is probably no easier target than Herbert Spencer (1820–1903). In colorful, attention-grabbing language, Spencer sometimes expressed contempt for the dissolute and dishonest elements within the lower socioeconomic classes, elements for whose lot there could be only one remedy. “If they are sufficiently complete to live, they do live, and it is well they should live. If they are not sufficiently complete to live, they die, and it is best they should die.”[12]

But even for Spencer, libertarian political institutions were justified on the basis of their ability to serve the common interests of all classes, not just the wealthy. In a fascinating exchange with the socialist H.M. Hyndman, for example, Spencer conceded that their disagreements were largely about means, not ends.[13] And even regarding means, Spencer was more willing than is generally recognized to call for acts of positive beneficence and concern for the poor when doing so was necessary to prevent unnecessary suffering—that is, suffering that served no educative or other salutary purpose. Men who suffer as a result of “accidents” and “unforeseen events, men who have failed for want of knowledge inaccessible to them, men ruined by the dishonesty of others, and men in whom hope long delayed has made the heart sick” ought to be helped. And “even the prodigal, after severe hardship has branded his memory with the unbending conditions of social life to which he must submit, may properly have another trial afforded him.”[14] Those misled by the popular description of Spencer as a “Social Darwinist” are likely to miss the key fact that, for Spencer, the overriding moral goal was to facilitate the evolution of society to a point where the interests of all persons are harmonized. Capitalism is not about the triumph of the strong over the weak; it is about the triumph of all over a state of society in which the interests of some are advanced by predation against others.

But then what about the archetypes of libertarianism themselves—Mises, Rothbard, and Rand? Just how far did they deviate from the principles and concerns of their intellectual predecessors?

During the Progressive era, Mises complained that advocates of the New Liberalism “arrogate to themselves the exclusive right to call their own program the program of welfare.” Mises regarded this as “a cheap logical trick.” The fact that classical liberals do not rely upon direct, state-based programs to distribute benefits does not mean that they are any less concerned for the poor.[15] Defending his preferred system of economic liberty, Mises wrote: “Any increase in total capital raises the income of capitalists and landowners absolutely and that of workers both absolutely and relatively… The interests of entrepreneurs can never diverge from those of consumers.”[16] If capitalism benefits the poor not just in real terms but also relatively to the wealthy, then capitalism is especially beneficial to the poor.

Mises’s critics (and some of his defenders) read Mises as whitewashing an uncompromising system of economic liberty with the idle hope that such a system maximizes productivity. On this reading, it is overall productivity that Mises cared about, and the distributional pattern that results is something about which Mises cared not one jot. However, notice what Mises did not say. He did not say: “The institutions of commercial society generate the greatest aggregate wealth and so, even though such institutions predictably deposit 20 percent of the population in a position of hereditary inferiority, this is A-OK.” Instead, Mises thought capitalist institutions justified, at least in part, because he believed a society-wide system of voluntary exchange will be materially beneficial for all citizens. Inequalities are justified, Mises seems to have argued, at least in part because they work to the material benefit of the least well off.

Indeed, Mises was explicit about the normative role he saw such claims playing within his defense of the free society. Thus: “In seeking to demonstrate the social function and necessity of private ownership of the means of production and of the concomitant inequality in the distribution of income and wealth, we are at the same time providing proof of the moral justification for private property and for the capitalist social order based upon it.”[17] The social function of inequalities—the benefits they provide to all, especially the poor—is an essential element in their moral justification.

It is no surprise, therefore, that when describing man’s role as a member of a (properly) liberal social order, Mises declared that each person “must adjust his conduct to the requirements of social cooperation and look upon his fellow men’s success as an indispensable condition of his own.”[18] Society, according to Mises, is a cooperative venture for mutual gain. In a good and just social order, people look upon the special talents of the fellow citizens not as weapons to be feared but as in some sense a common bounty. Economic competition is a morally praiseworthy form of social cooperation at least in part because it channels the talents of each towards the production of benefits for all.

Rothbard’s moral appraisal of capitalism was far more philosophical than that of his mentor, Mises. And Rothbard was sometimes insistent on the point that concerns about the welfare of the poor played no formal role in the moral justification of a free market. The fact that respect for natural rights yields economic prosperity that benefits all is a “fortunate…result.” But even if some other system could be shown to be more productive, the libertarian would still defend capitalism as the only moral economic system.[19] And what if libertarianism were not only less productive than rival economic systems, but actually led to the results sometimes forecast by Mises’ intellectual opponents? What if it led to the immiseration and exploitation of the working classes? Rothbard did not address this possibility explicitly, but the logic of his position suggests that this too would be irrelevant.

Of course, Rothbard did not actually think that free markets were bad for the poor. Quite the opposite. “The advent of liberty,” he wrote, “will immeasurably benefit most Americans.” If the government would get out of the way, the productive energies of citizens of every economic stratum would be unleashed. According to Rothbard, “the result will be an enormous increase in the welfare and the standard of living of everyone, and most particularly of the poor who are the ones supposedly helped by the miscalled ‘welfare state.’” A reduction in taxation, Rothbard emphasized, “would benefit the lower income groups more than anyone else.”[20] The only ones who would lose would be the politicians and crony capitalists who had been feeding at the public trough.

As for Ayn Rand, few would accuse her of being a bleeding heart libertarian. And yet, even here, and especially in her novels, we find something like a commitment to the norms of public justification and concern for the poor.[21] In Atlas Shrugged, Rand’s hero John Galt leads the productive and talented members of society to go on strike as a protest against the politics of increasing socialism and the morality of altruism that, in her view, underlies it. But nowhere in this novel do we find the suggestion that socialism is actually good for the poor. Instead, Rand presented us with a world where socialist economies are independently collapsing of their own weight. John Galt’s strike in the United States merely hastens the inevitable. And who suffers from these collapses? Not the talented. Precisely because of their productivity, they are able to build happy lives for themselves in the political isolation of Galt’s Gulch. The person who suffers is Eddie Willers—the average man of no exceptional talent. Willers suffers most from the collapse of society in Rand’s novel precisely because he is dependent on the productive for his survival and flourishing. More precisely, we might say, the average man depends on a social system that draws forth the productive energies of the talented in a way that benefits him and society at large.

Is this a part of the justification of capitalism for Rand? Not in her explicit philosophical writings. In those writings, Rand, like Rothbard, seems to have taken the view that the sole justification of laissez-faire is its respect for the natural rights of man. The fact that capitalism benefits the least well-off is just a happy coincidence. It’s a nice thing, perhaps, that we live in a world where the interests of the poor are not in conflict with the demands of morality. But the fact that capitalism serves the interests of the poor plays no role whatsoever in its moral justification.

What distinguishes the postwar libertarians in our view, what makes them in many ways the paradigms of libertarianism, is their enthusiasm for synthesis and systematization. But system-building has its costs. One of the costs is that insights that do not fit neatly into one’s axiomatic worldview are discarded. The history of prewar free market thinking, we contend, contains moral resources that are worth recovering—even at the cost of conceptual simplicity. The classical liberal tradition of Locke, Smith, and Spencer, for example, gave great weight to property rights in a way that checked governmental power. But none of the early liberal thinkers treated property rights as moral absolutes, and thus none of them was forced by axiom to deny that concern for the poor was a legitimate consideration in institutional design. We believe that classical liberalism, not axiomatic libertarianism, is the true heir of the liberal tradition.

Our impatience with axiomatic deductions does not require a rejection of principled argument, of course. It does not require that we discard the ideas of rights and duties and desert in favor of an anemic consequentialism. Hayek and Friedman themselves, perhaps, too often waved their hands at a naïve form of consequentialism when sophisticated moral analysis was called for. And their arguments often contained loose ends, not fully explaining why they allowed some market interventions but not others and thus lending an ad hoc air to their libertarian positions. But none of this shows that a sophisticated and principled philosophical analysis cannot be provided, only that Friedman and Hayek didn’t do it (and who can blame them?). And recent work in neoclassical liberalism illustrates the varied forms that principled analysis of this sort might take.[22]

Likewise, free marketers should not be afraid to express a principled concern for the poor, or even to commit themselves to an ideal of social or distributive justice. First, in its philosophically most sophisticated formulations, such as that of left liberal paragon John Rawls, social justice concerns the material condition of the lowest paid workers—not that of idle surfers, coffeehouse Marxists, the unemployable, or even the temporarily unemployed. Second, social justice is not a property of the particular distributions that emerge in a society but of social and economic institutions viewed as integrated wholes. Thus a commitment to social justice in no way commits one to advocating liberty-limiting “corrections” of emergent distributions on an ongoing basis.

We believe that Robert Nozick’s critique of “patterned” conceptions of justice obscured this point. But this is the feature of social justice that led Hayek, in our view correctly, to state that his differences with Rawls about social justice were “more verbal than substantive”.[23] Third, and as a consequence, a commitment to social justice does not require that one advocate “big state” welfare programs or anything even close. A set of institutions might well satisfy the requirements of social justice without including any state-based “redistributive” apparatus whatsoever.[24] After all, what are these requirements of social justice? According to Rawls, social justice allows for material inequalities, even extremely large and growing inequalities, provided only that the overall system works in a way that is beneficial to the lowest paid workers (that is, if the lowest paid workers in capitalist societies, over time, tend to earn more than the lowest paid workers in any noncapitalist alternative, then capitalist societies are better from the perspective of social justice).

This, then, is the gold standard of contemporary theorizing about social justice. The great historical defenders of commercial society such as Locke and Smith were all writing before social justice had been adequately defined. But seeing the theory of social justice now at hand, Locke and Smith would not hesitate to send out their free market institutions to battle the socialists and welfare statists for that crown. Inspired by their example, today’s bleeding heart libertarians stand ready to pick up their historical standard, and to carry it forth. Property rights, limited government, and social justice. Why not?

Notes

[1]According to Jan Narveson, for example, “[it] is plausible to construe all rights as property rights.” The Libertarian Idea, Philadelphia: Temple University Press, 1988, p. 66.

[2] See Robert Nozick, Anarchy, State, and Utopia, pp. 149-50, and Friedrich Hayek, Law, Legislation and Liberty, volume 2: The Mirage of Social Justice, pp. 62-63.

[3] We are calling attention to the shared institutional perspective of these thinkers, while noting that they differ in the moral foundations of their view (Mises, for example, argues primarily from a principle of economic efficiency rather than self-ownership).

[4] When the first meeting of the Mont Pelerin Society, a group that included Hayek and Milton Friedman, included a discussion about whether a progressive income tax could ever be justified, an unhappy Mises declared “You’re all a bunch of socialists!” Brian Doherty “Best of Both Worlds,” Reason.com, June 1995.

[5] On neoclassical liberalism, see Jason Brennan and John Tomasi, “Classical Liberalism,” forthcoming in David Estlund, ed., Oxford Handbook of Political Philosophy (available online here. For an example of a neoclassical perspective on the specific issue of liberty, see the Cato Unbound debate on “Conceptions of Freedom,” March 2010.

[6] Matt Zwolinski is the founding editor of the blog Bleeding Heart Libertarians. John Tomasi is the author of Free Market Fairness, New York: Princeton University Press, 2012.

[7] The working title of our book is Libertarianism: A Bleeding Heart History, under contract with Princeton University Press.

[8] See Gerald Gaus, The Order of Public Reason. A good short introduction is his recent essay “The Range of Justice (Or, How to Retrieve Liberal Sectual Tolerance),” Cato Unbound, October 2010.

[9] Even co-authors rarely agree on everything. Regarding Locke’s (so-called!) proviso, see John Tomasi “The Key to Locke’s Proviso” British Journal for the History of Philosophy 6/3 1998: 447-454.

[10] Two Treatises of Government, II, sect. 41. For a discussion of the “bleeding heart” elements in Locke’s thought, see this essay.

[11] Wealth of Nations, New York: Classic House Books, 2009, 60.

[12] Herbert Spencer, Social Statics, chapter 18.

[13] The Man versus the State Bibliolife, 2009 (1884), 43.

[14] Social Statics, Chapter 25. For more on Spencer and the poor, see here and here.

[15] Human Action: A Treatise on Economics Indianapolis, IN: Liberty Fund, 2007, 834.

[16] Liberalism New York: The Foundation for Economic Education, 1985 164-5.

[17] Liberalism, 33.

[18] Liberalism, 14.

[19] For a New Liberty Auburn, Alabama: The Mises Institute, 2006 (1973), pp. 48-9.

[20] For a New Liberty, p. 202-3.

[21] On the missing morally callous libertarians, see here.

[22] In addition to the broadly Hayekian/Rawlsian fusionist approaches taken by Gaus and Tomasi, for instance, some neoclassical liberals such as David Schmidtz have argued on the grounds of a principled moral pluralism. See for instance, his Elements of Justice (Cambridge University Press, 2006). For a pluralistic account of liberty in particular, see Schmidtz and Brennan, A Brief History of Liberty (Blackwell, 2010).

[23] Law, Legislation and Liberty, vol. 2 The Mirage of Social Justice (Chicago: University of Chicago Press, 1977), xiii, see also 74, 100. For analysis of Hayek’s view, and for an exposition of the other points made in this paragraph, see Tomasi, Free Market Fairness.

[24] The path-breaking essay is Daniel Shapiro, “Why Rawlsian Liberals Should Support Free-Market Capitalism,” Journal of Political Philosophy 3/1 (1995): 58-85.

    Response Essays

    In Praise of Bleeding Heart Absolutism

    In their thought-provoking piece, Matt Zwolinski and John Tomasi (henceforth Z&T) argue that the most prominent version of contemporary libertarian thought — that represented by the Unholy Trinity of Mises, Rand, and Rothbard (I’m not sure why Z&T don’t include Nozick, since most of what they say about the first three applies much more obviously to him) — departs from the historical tradition of classical liberalism in a number of respects that are largely to the newer version’s disadvantage.

    Pinning down precisely what Z&T take the differences to be (as well as in what respects they take them to be defective) is surprisingly difficult, however. Here’s a list of some of the points to which they appeal:

    a) whether property rights are based on self-ownership

    b) whether property rights are absolute or nearly so

    c) whether the requirements of justice are exhausted by individual property rights

    d) whether the concept of social justice is to be rejected

    e) whether “the moral justification of free market institutions is logically independent from any claims about the effects of those institutions on the material holdings of the poor”

    f) whether a “significant role for the state in the provision of a social safety net” is to be rejected

    But I worry that Z&T run some of these together, or seem to, or at least write in such a way as to make readers likely to do so; so it’s important to see that they’re distinct. One could say yes to (a) but no to (b), for example, if one thought that self-ownership was easily overridable; one could say no to (c) but yes to (d), if one thought (as, e.g., Rand did) that justice was a principle of interpersonal morality broader than respecting rights; and one could say no to (e) but yes to (f) if one thought that the poor would do better in a genuinely freed market than under a welfare state.[1] (This latter point is one that Z&T eventually stress toward the end of their essay, but in the opening section it would be easy to get the impression that they are urging a negative answer to (e) and thus to (f).)

    In any case, it’s not entirely clear what distinction is intended to be drawn by invoking the factors on this list. The initial impression we get is that the Unholy Trinity (Mises, Rand, and Rothbard) all answered yes to these questions while older liberal thinkers generally answered no; but Z&T are quick to admit that this is not actually so. Noting in a footnote that Mises “argues primarily from a principle of economic efficiency rather than self-ownership” and so does not fit their categories, they explain that they are “calling attention to the shared institutional perspective of these thinkers” rather than the “moral foundations of their view.” But the only items on the list that concern institutions are (e) and (f), and Z&T themselves acknowledge that while Mises says yes to (f), he says no to (e). So it’s a mystery how Mises gets inducted into the Unholy Trinity in the first place. In any case, if Z&T are really concerned only with institutional perspectives rather than moral foundations, why do they bring up the other items on the list in the first place? What precise set of issues is it that is supposed to divide the sheep from the goats here?

    I don’t think their characterisation of the other two members of the Unholy Trinity is quite right either. While admitting that Rand and Rothbard agree with Mises that a “society-wide system of voluntary exchange will be materially beneficial for all citizens,” Z&T insist that this fact plays no role in moral justification; for Rand and Rothbard, “the sole justification of laissez-faire is its respect for the natural rights of man. The fact that capitalism benefits the least well-off is just a happy coincidence.”

    This characterisation of their views is radically incomplete, however, inasmuch as it leaves out the fact that, for both Rand and Rothbard, natural rights are themselves grounded on considerations of benefit. This is famously true of Rand, who based her ethics on the need of each person to seek his or her own benefit, such benefit being defined in Aristotelean fashion as that which furthers one’s own life as a rational being. But it is no less true of Rothbard, likewise an Aristotelean, for whom natural rights are based on natural law, which in turn is a “science of happiness” that “elucidates what is best for man — what ends man should pursue that are most harmonious with, and best tend to fulfill, his nature.”[2]

    Neither Rand nor Rothbard, then, is a strict deontologist of the Nozickian sort; both of them ground natural rights in an ethic that is understood to be beneficial to each person who practices it, whether rich or poor. When Rothbard uses the “happy coincidence” language, he is referring to additional benefits beyond those already invoked in determining our natural rights. (Admittedly the “happy coincidence” terminology is somewhat unfortunate, especially if one holds as I do that moral theory should be driven by both deontological and consequentialist considerations, each serving to shape the contours of the other via mutual adjustment, so that their final convergence will be no coincidence; nevertheless, the term does capture the familiar but striking phenomenon that deontological and consequentialist considerations tend to converge to a considerable degree already, even prior to mutual adjustment.)[3] Thus while none of the Unholy Trinity places special emphasis on benefit to the poor, all three not only regard libertarianism as beneficial to all people (including the poor), but they also make use of such benefit in their justification of libertarian principles — a fact which seems to me to weaken some of the distinctions that Z&T are attempting to draw.

    It’s also worth pointing out that Rothbard revived, and made central to his political theory, the class analysis approach pioneered by such 19th-century liberals as Thomas Hodgskin, Augustin Thierry, and Lysander Spooner, that focused on the ways in which wealthy elites owe their position to government intervention at the expense of the less affluent (thus making differential access to state power, rather than differential access to the means of production, the basis for class rule). As Rothbard writes:

    [W]hile big business would indeed merit praise if they won that bigness on the purely free market … in the contemporary world of total neo-mercantilism and what is essentially a neo-fascist “corporate state,” bigness is a priori highly suspect, because Big Business most likely got that way through an intricate and decisive network of subsidies, privileges, and direct and indirect grants of monopoly protection.[4]

    Any discussion of Rothbard’s concern for the less affluent is surely incomplete without a recognition of the class-analysis dimension of his thought.

    Z&T point to Milton Friedman and Friedrich Hayek as more reasonable alternatives to the Unholy Trinity — more consequentialist and friendlier to state action. While this is broadly accurate, it must be remembered that neither of them was a pure consequentialist in any straightforward sense; Friedman wrote that “Desirable or not, any end that can be attained only by the use of bad means must give way to the more basic end of the use of acceptable means,”[5] and Hayek’s Law, Legislation, and Liberty is essentially a book-length statement of the same principle. It’s likewise worth noting that Friedman and Hayek both became increasingly antistatist over the course of their careers (Hayek to the point of saying that if he were younger he would be an anarchist).[6] Certainly there are important differences between Friedman and Hayek on the one hand and the Unholy Trinity on the other (just as there are important differences within each grouping), but these differences do not seem to line up neatly with Z&T’s categories.

    I also find unconvincing Z&T’s attempts to draw sharp distinctions between contemporary libertarians influenced by the Unholy Trinity on the one hand, and libertarianism’s pre-20th-century precursors on the other, with the latter friendlier to social justice and less friendly to absolutist property rights. If the claim is simply that, say, 19th-century classical liberals were often more “left-wing” — more explicitly aligned with such causes as feminism,[7] antiracism, and the labor movement[8] — and that contemporary libertarians would do well to follow them in these respects, then I certainly agree. But I cannot follow Z&T when they say that “none of the early liberal thinkers treated property rights as moral absolutes, and thus none of them was forced by axiom to deny that concern for the poor was a legitimate consideration in institutional design.”

    First: I’m not sure whether treating property rights as moral absolutes means treating them as ungrounded or treating them as non-overridable (or at least nearly so). It’s true that that no early liberal thinkers treated property rights as ungrounded, but no contemporary libertarian thinker does so either. And if “absolute” instead means non-overridable or nearly so, then there are surely quite a few property absolutists among early liberal thinkers, including Hodgskin, Spencer, Bastiat, Molinari, and Spooner. The historical distinction that Z&T are trying to draw is one I simply don’t find sustainable by the historical evidence.

    Second: the phrase “and thus none of them was forced by axiom to deny …” implies that if one does treat property rights as non-overridable or nearly so, then one is necessarily prevented from treating concern for the poor as “a legitimate consideration in institutional design.” Yet once again, this ignores the possibility (despite their acknowledgment of its actuality in discussing Mises and Spencer) that concern for the poor might itself be part of the grounding for the absolute property rights that are in turn appealed to in institutional design.

    I’m puzzled by Z&T’s treatment of several specific historical figures as well — namely Locke, Smith, and Spencer.

    With regard to Locke, I find especially baffling Z&T’s attempt to read contemporary public-reason liberalism into his thought. Yes, Locke certainly upheld the “essential freedom and equality of human beings,” with its corollary rejection of any “natural authority or subordination among men.” But where on earth do they find in Locke the claim that “any authority or subordination that emerges — including the kinds involved in enforcing a system of property rights — must be justifiable to each man by his own lights”? It seems to me that several different aspects of Locke’s theory are being run together.

    On the one hand, the institution of the state must rest on the consent of those who are to be governed by it, precisely because the establishment of the state involves the suppression of competition (which Locke — mistakenly — thinks is required for the suppression of the right to be a judge in one’s own case); but here the consent required is actual consent, not the hypothetical consent implied by the public-reason use of “justifiable.” On the other hand, the kind of force used simply in defense of person and property, whether by individuals or by the state, does not for Locke require justification, either actual or hypothetical, in terms of each individual’s “own lights”; it is justified in terms of natural law, and if natural law diverges from some people’s “own lights,” so much the worse for the latter. For Locke, it is precisely because there is no natural subordination among people that it is legitimate to use force against those who seek to subordinate others by aggressing against their persons or property.

    As for the Lockean Proviso, this is grounded on property rights — the residual property rights of the human community, inheritors of God’s grant to humankind of dominion over the earth — rather than on straightforwardly consequentialist grounds; and here again the constraint is that people’s actual rights under natural law be respected, not that their possibly mistaken “own lights” be so.

    Z&T might also have mentioned in support of their case Locke’s apparent endorsement (I say “apparent” because the precise interpretation of the passage is controversial) of welfare rights at First Treatise of Government IV.42; but Locke’s reason for endorsing welfare rights is that in this way the affluent are prevented from making themselves masters over the destitute — so this positive right is grounded in a negative right not to be subordinated, not in anything like a public-reason approach. (Similar remarks apply to Spencer’s treatment of land rights at Social Statics IX.)

    With regard to Smith, while there are certainly consequentialist aspects to his thought, they should not be overemphasized. David Hume had argued that the value of justice lay in its effects on society as a whole.[9] Smith explicitly rejected Hume’s account, maintaining instead that although we sometimes “have occasion to defend the propriety of observing the general rules of justice by the consideration of their necessity to the support of society,” yet it is really the “intrinsic hatefulness and detestableness” of unjust acts “which originally inflames us against them,” inasmuch as “they are the natural and proper objects of hatred and detestation”; and we find ourselves driven to appeal to the social consequences of injustice, rather than its intrinsic wrongness, only when we are trying to convince people who, through “the corruption [or] vanity of their hearts,” are unmoved by the greater reasons and so must be met with the lesser ones.[10] Rather Rothbardian sentiments, these.

    Smith further distinguishes sharply between the negative virtue of justice and the positive virtue of beneficence: “Beneficence, therefore, is less essential to the existence of society than justice.”[11] A “violation of justice … does real and positive hurt to some particular persons” and is thus “the proper object of resentment, and of punishment.”[12] Justice is a “negative virtue” that “only hinders us from hurting our neighbour.” Thus someone who “abstains from violating either the person, or the estate, or the reputation of his neighbours,” Smith tells us, “fulfils … all the rules of what is peculiarly called justice, and does every thing which his equals can with propriety force him to do, or which they can punish him for not doing. We may often fulfil all the rules of justice by sitting still and doing nothing.” The “mere want of beneficence,” by contrast, merits “no punishment from equals.”[13] It may legitimately be punished by a superior, and thus the “civil magistrate” is empowered to “prescribe rules, therefore, which not only prohibit mutual injuries among fellow-citizens, but command mutual good offices”;[14] here indeed we leave Rothbardian territory. But what the civil magistrate commands is still beneficence, not justice, and his right to command it is grounded, for Smith, not on human equality but on inequality.

    With regard to Spencer, the contrast that Z&T draw between his work and that of the Unholy Trinity is especially puzzling, given that Spencer seems so close a precursor to their perspective. Did Rothbard derive all of justice from a single libertarian principle? So did Spencer. Did Spencer allow considerations of benefit to shape that principle? So, as we’ve seen, did Rothbard. Did Spencer acknowledge the existence of positive (non-enforceable) charitable obligations over and above negative individual rights? Again, so did Rothbard.

    Incidentally, despite their overall defense of Spencer, I am sorry to see Z&T fall in with the long and shameful practice by Spencer-bashers of quoting Spencer’s infamous line about the poor (“If they are sufficiently complete to live, they do live, and it is well they should live. If they are not sufficiently complete to live, they die, and it is best they should die.”) while omitting the first sentence of the immediately following paragraph (“Of course, in so far as the severity of this process is mitigated by the spontaneous sympathy of men for each other, it is proper that it should be mitigated.”),[15] thus giving the impression that Spencer is advocating letting the poor die off when the whole point of the passage is to explain why he’s not advocating that. As they rightly point out, Spencer believes we have duties of beneficence (both positive and negative) in addition to our duties of justice. This didn’t stop Spencer from having a highly “axiomatic” approach to justice, an “absolutist” approach to property rights, and a strong prohibition against governmental regulation and welfare programs.

    And if being in favor of charity makes Spencer a bleeding-heart libertarian, then Rand was a bleeding-heart libertarian too; after all, it was Rand who wrote that the “idea of hardships being good for character and of talent always being able to break through is an old fallacy,” since “success requires both talent and luck,” and “the ‘luck’ has to be helped along and provided by someone.” Such help “not only provides human, decent living conditions which a poor beginner could not afford anywhere else,” Rand explains, but also “makes a beginner feel that he is not, after all, an intruder with all the world laughing at him and rejecting him at every step, but that there are people who consider it worthwhile to dedicate their work to helping and encouraging him. Isn’t such an organization worthy of everyone’s support?”[16]

    Again, I don’t mean to deny that there are interesting differences between Unholy-Trinity-style libertarianism and its pre-20th-century precursors; but these differences are complicated, cutting in various different directions, and do not seem to align in any straightforward way with the distinctions that Z&T are inviting us to draw.

    While insisting that a concern with consequences or social justice need not mean a readiness to accept a greater role for the state, Z&T nevertheless point, as a model for future libertarian theorizing, to contemporary libertarian thinkers like David Schmidtz and Gerald Gaus, who, whatever their (in fact considerable) merits, are surely friendlier to state action than are the types of libertarian that Z&T mean to criticize — thus reinforcing the impression that to accept the moral upshot of Z&T’s arguments is to become less intransigent in opposing state power. Hence it’s worth pointing out that there are plenty of contemporary libertarian thinkers who place considerable emphasis on consequences (e.g., Randy Barnett, Jan Narveson, and our fellow symposiast David Friedman), or on social justice and the needs of the poor (e.g., Kevin Carson, Gary Chartier, and Charles Johnson),[17] who nevertheless end up with far more thoroughgoingly antistatist positions than, e.g., Mises or Rand.

    With regard to moral foundations, I concur with Z&T that considerations of benefit must play a role in shaping the requirements of justice; but I think it must also be a two-way street. I am firmly in agreement with what I regard as the crucial insight of classical Greek ethics, namely that justice is an internal, not just an external, means to human welfare, and that is one reason that I cannot follow Z&T in regarding the neo-Rawlsian approach as the “gold standard of contemporary theorizing about social justice”; it is too one-sided. Rawlsians, like utilitarians, take the content of human benefit as given (whether objectively or subjectively) apart from the content of justice, and then try to shape justice in accord with it. But ever since Plato showed that no one counts as just unless she regards justice as constitutive of, not merely as causally conducive to, human welfare, we’ve all known, or should have known, that considerations of justice must play a role in shaping our conception of what counts as a benefit. Justice and benefit stand in mutual determination.[18]

    Another of my reasons for not embracing the neo-Rawlsian approach as the “gold standard of contemporary theorizing about social justice” is that it is deficient in class analysis and lacks a theory of exploitation. It’s odd that in documenting 19th-century liberals’ concern for the poor, Z&T say nothing about the 19th-century liberal appeal to class-based exploitation as an explanation of the plight of the poor — especially since one of them (Zwolinski) has written sympathetically on this topic elsewhere.[19]

    I think 19th-century liberal class theory, and its revival by Rothbard and others, offer a more compelling approach to social justice than the relatively anemic Rawlsian variety. As I’ve written elsewhere:

    [T]he advantage, as I see it, of the historical challenge to existing inequalities is that it lays bear the class structure of society, and the roots of such inequalities in state violence. Merely pointing to the fact that some people have a lot more than others is less compelling as a critique; it invites the response “So what? Those who have more aren’t hurting anybody; you’re just appealing to envy.” By contrast, being able to show that those who enjoy a higher socioeconomic status have to a considerable extent achieved and maintained that status by forcibly expropriating and oppressing the less affluent provides for a far more effective indictment.

    I don’t mean to be claiming merely that appeal to the historical approach is more rhetorically effective … My point is that the historical critique correctly identifies what is surely a morally relevant fact, and one that end-oriented critiques tend to ignore: namely, that in many, many cases those who have more are getting it at the expense of those who have less. Now of course utilitarian and Rawlsian approaches may also make the claim that, in some sense, those who have more have it at the expense of those who have less; but in order to substantiate that claim without appealing to historical (i.e. causal) considerations, they have to defend a baseline of equality. My present argument is not that such a defense is impossible, but only that the need to defend it places an additional and somewhat recondite burden on end-oriented challenges to inequalities — whereas the historical challenge, by identifying past and ongoing acts of violent expropriation rather than merely pointing to the existence of differential shares, provides a much more straightforward, intuitive, and unambiguous basis for condemning the present structure of wealth distribution in “capitalist” society.[20]

    In conclusion, I think that both classical liberalism and contemporary libertarianism, at their best, exhibit simultaneously a commitment to social justice and a more or less absolutist, self-ownership-oriented theory of property rights. We don’t have to choose: we can be bleeding-heart absolutists.

    Notes

    [1] For my own views on the concept of social justice, see my “Proletarian Blues.”

    [2] Murray N. Rothbard, The Ethics of Liberty (New York University Press, 1998), p. 12.

    [3] For fuller discussion, see my “Why Does Justice Have Good Consequences?

    [4] Quoted in Peter Klein, “Rothbard on Big Business.”

    [5] Milton Friedman, Capitalism and Freedom (University of Chicago Press, 2002), p. 22.

    [6] Sudha R. Shenoy, “The Global Perspective.”

    [7] See my “Libertarian Feminism: Can This Marriage Be Saved?,” co-authored with Charles Johnson.

    [8] See, for example, Herbert Spencer’s call for workers’ cooperatives to replace hierarchical capitalist firms at Principles of Sociology VIII.20.

    [9] David Hume, Treatise of Human Nature III.ii.

    [10] Adam Smith, Theory of Moral Sentiments II.ii.22-23.

    [11] Ibid., II.ii.17.

    [12] Ibid., II.ii.5.

    [13] Ibid., II.ii.9.

    [14] Ibid., II.ii.8.

    [15] Herbert Spencer, Social Statics XXVIII.4.

    [16] Ayn Rand, Letter to Marjorie Williams (18 June 1936); in Michael S. Berliner, ed., Letters of Ayn Rand (Dutton, 1995), pp. 31-33.

    [17] For these latter, see Gary Chartier and Charles W. Johnson, eds., Markets Not Capitalism: Individualist Anarchism Against Bosses, Inequality, Corporate Power, and Structural Poverty (Minor Compositions, 2011).

    [18] For further details, see “Why Does Justice Have Good Consequences?,” op. cit.

    [19] See my “Comments on Ruth Sample’s ‘Is the State Exploitative?’ and Matt Zwolinski’s ‘Toward a Theory of State Exploitation’.”

    [20] “Left-Libertarianism, Class Conflict, and Historical Theories of Distributive Justice.”

      Natural Rights + ?

      In their lead essay, Matt Zwolinski and John Tomasi combine two different claims, one historical, one philosophical. The historical claim is that the earlier thinkers in the classical liberal/libertarian tradition were less sympathetic to a hard line propertarian version of their position, more sympathetic to one in part based on concepts of social justice, than postwar libertarian thinkers such as Rand and Rothbard. The philosophical claim is that the earlier thinkers were correct—that a version of libertarianism incorporating elements of social justice, in particular a special concern for the deserving poor, is what modern libertarians ought to aim at.

      I agree that the version of libertarianism they criticize does not fully reflect the views of earlier thinkers in the classical liberal/libertarian tradition. I also agree that, considered on its own merits as a moral theory, it is in several respects unsatisfactory. But I disagree with the authors’ view of what is missing, both as a historical account and as a moral theory. What the hard line propertarian version leaves out is not social justice but human welfare. A more nearly correct version of libertarianism would owe more to Bentham than to Rawls.

      Of the early writers, Smith is the one with whom I am most familiar. When he writes that “It is but equity, besides, that those who feed, clothe and lodge the whole body of the people, should have such a share of the produce of their own labour as to be themselves tolerably well fed, clothed and lodged,” he is appealing not to egalitarian sentiments but to producers getting a fair share of what they produce. Insofar as any concept of justice is implicit in that quote it is closer to Rand than to Rawls.

      What is there in his view beyond a concern with people getting what they are entitled to? Our lead authors’ repeated references to “the poor,” in an essay written by and for moderns, badly misrepresents the 18th century world and Smith’s view of it. When Smith was writing, the working class, the people Smith is referring to in that quote, represented not the lower end of the income distribution but the bulk of the population.

      Thus, earlier in the same paragraph, he writes: “Servants, labourers, and workmen of different kinds, make up the far greater part of every great political society. But what improves the circumstances of the greater part can never be regarded as an inconvenience to the whole. No society can surely be flourishing and happy, of which the far greater part of the members are poor and miserable.” That looks more like an argument based on something like (pre-Benthamite) utilitarianism than one based on social justice.

      One can get a clearer idea of Smith’s view of issues of equality from his long discussion of alternative forms of taxation. He begins with a set of maxims, of which the first is:

      I. The subjects of every state ought to contribute towards the support of the government, as nearly as possible, in proportion to their respective abilities; that is, in proportion to the revenue which they respectively enjoy under the protection of the state.

      While he rejected taxes directly on income (except, possibly, the income of government employees), he thought that the overall incidence of taxation ought to be in proportion to income. He was, in modern terms, a flat taxer. Thus he argues for taxing the luxuries of the poor rather than their necessities in order to be sure that the poor pay their fair share of the expenses of government, since he believed that a tax on necessities will result in a rise in wages, hence be ultimately paid by the not-poor. He is not offering a Randian or Rothbardian argument from the innate justice of property rights, but neither is he offering anything close to a modern view of “social justice.”

      Should he be? That brings us to the second half of the argument, disavowed at the beginning of the essay but returning at the end, the claim that the authors’ variant of libertarianism is superior to the alternative they reject. They write:

      Likewise, free marketeers should not be afraid to express a principled concern for the poor, or even to commit themselves to an ideal of social or distributive justice.

      One problem with the version of libertarianism exemplified by Rand, Rothbard, and their followers is its lack of any logical foundation sufficient to persuade the unbeliever of its strong claims. Another is its failure to answer many of the important questions, especially where to draw lines. And another is that, taken literally, it sometimes gives the wrong answer: I cannot turn on the lights in my house without prior permission from every landowner whose ability to see them demonstrates that my photons are trespassing on his property, and if I fall out my apartment window and end up clinging to a projecting flagpole a floor lower, I must let go and fall to my death when ordered to do so by its owner.

      But substituting “an ideal of social or distributive justice” is hardly an improvement, judged by either foundations or implications. Rawls’ derivation for his idea of social justice, the version that Tomasi and Zwolinski apparently wish to incorporate in libertarian thought, starts with the claim that someone facing a set of alternatives whose probability distribution is unknown—the imaginary social chooser behind a veil of ignorance—will act on the assumption that he is certain to end up with the least attractive possible outcome. That, plus a lot of hand waving, is all the justification for his “philosophically most sophisticated” version of social justice that I have been able to find.

      And one implication of that version, taken as literally as I have been taking the natural rights alternative, is that it is better to have a world where everyone is at a utility level of a hundred than a world with one person at ninety-nine and everyone else at a thousand. I have never yet been able to figure out why anyone takes either the derivation or the conclusion seriously.

      Tomasi and Zwolinski have written an essay of more than three thousand words on the history of classical liberal/libertarian thought without ever mentioning a central fact of that history, the close historical link between classical liberalism and utilitarianism. They have gone on to defend a version of libertarianism derived not from that history but from current fashions in academic philosophy, fashions that strike me as rather less persuasive than the utilitarianism of Bentham and Mill, despite that philosophy’s well-known problems.

      The version of libertarianism that seems most plausible to me is one where respecting rights is seen as a good thing, a value in itself as well as a means to other values, but not as a value that trumps all others. One reason to respect natural rights is that it is a good thing to do, another is that respecting them can be expected to produce a healthier, wealthier, and happier world than violating them.

      Utilitarianism does not, in my view, fully capture the range of those other values, but it comes considerably closer than social justice. I do not have an adequate derivation for my ethical views and—unlike Rand and Rawls—I know that I do not, so can only report on my moral intuitions while trying, so far as possible, to think through their implications and interrelations.

      One conclusion is that I can imagine circumstances where the consequentialist benefits of some act are sufficiently large relative to the cost in rights violation that I would approve of it—stealing a nickel from its rightful owner to prevent an asteroid strike that would destroy the world was my old example. Another is that I can imagine circumstances where the rights violation costs are sufficiently large relative to the utility gains so that I would disapprove of it—for an example, check the index of the second edition of The Machinery of Freedom (available as a free pdf from my web page) for the entry “utilitarian, why I am not.” The implications of my moral intuitions are not as tidy as the theories of Rand or Rawls or, for that matter, Bentham. But then, I know of no a priori reason to expect the truth, in moral philosophy or anything else, to always be simple.

      Let’s Reject the Purity Test

      All too often, individuals of the libertarian persuasion get caught up in debates over who is “more libertarian” than the other, or who is “actually libertarian.” The desire to “out-libertarian” one another can be a fun intellectual experiment born from a desire to take pride in our identity in the face of constant attack by others. And on the surface, it seems like this debate is of serious philosophical merit because it deals with the justification and practical application of the principles of liberty. In “A Bleeding Heart History of Libertarianism,” Matt Zwolinski and John Tomasi get caught up in this game. A common criticism of the Bleeding Heart Libertarian project is that it is not truly in the libertarian tradition because it does not defend libertarianism according to strict deontological grounds for property rights and because it potentially supports a social safety net and related public policies.

      Z&T play the game, arguing that the “benchmarks for ideological purity” of Mises, Rand, and Rothbard should be replaced by a new standard: “[T]he fate of the class who labor at the lowest end of the pay scale under capitalism is an essential element in the moral justification of that system.”[1] Since Z&T want push to the side the question of which strain of thought is “is more defensible on philosophic and other grounds,” I shall do the same. I will argue that, in terms of the history of libertarian thought, the ideological purity game Z&T have been pulled into is deeply misconceived and should be rejected.

      There are three components to a comprehensive political philosophy: justification, principle, and policy. The justification for a political philosophy is the standard used to ground one’s beliefs, such as maximizing the greatest good for the greatest number or respect for property rights. Principles are the mid-level values that articulate what a philosophy cares most deeply about. Policy is the practical application of those principles to specific, real-world problems. In daily political life, policy is the center of discussion and concern, dealing with questions like, “Should we raise or lower taxes?” The principles that underlie one’s policy positions sometimes come out when people ask, “Should we care more about following the restrictions of the Constitution or helping those in need?” Questions like this deal with the values people hold, which guide policy decisionmaking. The justification of those values is almost exclusively reserved for academic conversations (unfortunately), when people ask questions like, “Should liberty be of greater value than equality? By what standard can we make such a decision?”[2]

      The libertarian intellectual tradition includes a wide-ranging diversity of justifications for libertarian principles as well as heated debates over the proper policy prescriptions of libertarianism. From the perspective of justification, the tradition includes the consequentialism of J.S. Mill, the deontology of Robert Nozick, the contractualism of John Locke, the anti-contractualism of Lysander Spooner, the natural rights theory of Ayn Rand, and the social progress of F.A. Hayek. Even labeling these luminaries with a single term to describe their justificatory system seems to not do them justice because it oversimplifies and to a certain extent misrepresents their views. What is important is that they do not share a common justification with one another, or even with themselves at times (as Z&T point out in their original post), but they are all part of the libertarian tradition. There is no single justification of political philosophy that is “pure” libertarianism.

      Similarly, there are great divisions of opinion in the application of libertarianism to policy positions. There are open debates on topics like the death penalty, abortion, the ideal tax system, solutions to the damaging governmental education system in the United States, and so on. While many libertarians opposed the invasion of Iraq, Randy Barnett wrote a strong, libertarian defense of pre-emptive intervention.[3] While libertarians agree on things like the need for minimal government, there are many open debates on the specific policy prescriptions a minimal government would entail.[4]

      What unifies libertarian thought is a set of commitments to mid-level principles that espouse the importance of individual liberty for all people in all areas of their lives, both social and economic, which must be respected by institutions with the power to use force in both domestic and foreign affairs. Even articulating the fundamental principles at this level will encourage debate amongst libertarians. Does Cato’s articulation of “individual liberty, limited government, free markets, and peace” do it best? Or is it wrong to segment these principles into different areas since “free markets” and “peace” are just different facets of the singular principle of individual liberty? This may too be up for debate, but the fundamental commitment to individual liberty to all individuals, in all areas of a person’s life, provides a recognizable framework from which to discuss the meaning and implications of libertarianism.

      Z&T do not differentiate between justification, principles, and policy in their analysis of the consistency of various libertarian figures. This is why I agree with Roderick Long’s confusion over many of their interpretations and apparent misinterpretations of historical libertarian thinkers.[5] At times, Z&T argue that the purity test involves the justification for political philosophy, particularly with people like Locke, Mises, and Rand. At other times, Z&T argue in the realm of policy prescription, such as when dealing with the social safety net.[6] These are related but separate issues. The shift back and forth between them and the mid-level principles illustrates a natural confusion that comes with playing the ideological purity game.

      What’s more, the purity game relies on limiting the scope of libertarian thought to particular figures, as Z&T do with Locke and Smith. Yet the history of libertarian thought is much larger and more diverse than that. In The Libertarian Reader, David Boaz argues that “the main thread of libertarianism goes back to the Jewish and Greek idea of a higher law, a law by which everyone, even the ruler, could be judged. The simple idea that the will of the ruler was not the ultimate source of authority…”[7], long before Locke was writing. A sampling of authors Boaz cites as influential libertarians includes Isabel Paterson, Thomas Paine, Frederick Douglass, John Stuart Mill, and Frédéric Bastiat. It would be wrong to claim these figures were “less libertarian” because they dealt with different policy issues or offered different justifications for their beliefs during different time periods.

      The principles of libertarianism are what tie these diverse thinkers together and allow us to understand the development of their thought against the background of the intellectual landscape of their time. Locke was responding to Robert Filmer and Thomas Hobbes, and he drew upon all the tools in his arsenal to do so, including the independence of human beings from the authority of others: “every man has a ‘property’ in his own ‘person.’ This nobody has any right to but himself.”[8] Bastiat was responding to Rousseau, whom he criticized for seeing, “between the lawgiver and the rest of mankind as great a distance, or rather as great a gulf, as that which separates the inventor of the machine from the inert matter of which it is composed.”[9] Smith’s targeting of mercantilism continually drew his focus back to the nation as a whole and the strategies for supporting the nation.

      The 20th century was markedly different because socialism presented a new challenge to the philosophy of liberty. Prior justifications grounded in natural moral equality soundly defeated claims to the legitimacy of state-sponsored inequality. But socialist argument that legal equality was a threat to equality of outcome presented a new intellectual challenge that required a new response. Especially after the Second World War, the threat of socialism was liberty’s intellectual enemy number one. While Z&T describe this as “postwar” libertarian thought, a more apt description would be “Cold War Era Libertarianism.” Rand was driven in large part by an opposition to communism. Nozick was primarily concerned with John Rawls and anarchism.[10] A concern for the individual became paramount as the intellectual opponents of liberty focused on the all-encompassing power of the state to control society for the good of the collective.

      With the end of the Cold War and the start of the 21st century, libertarianism has had a chance to become more attuned to sources of power that threaten liberty other than communism and appeals to the collective. As a result, many young libertarians are paying more attention to the role that group identity plays in forming our lives and our liberty.[11] BHL is one, but certainly not the only, version of that. If we can glean a historical narrative of libertarianism from Z&T’s article, perhaps it would be the following trend at the level of justification for libertarianism:

      Society as a Whole –> Individual –> Individual within Society

      In other words, up through the 19th century, the primary justification for libertarian political philosophy could be found in the claims it made for social order and the average well-being of the individuals within society. In the Cold War era of libertarian thought, opposition to socialism led libertarians to an emphasis on individualism. With the end of the Cold War, the rise in threats to liberty from government intervention in areas other than economics, and the ability to study the effects of policy not just on the average individual in society, but those on the wider margins, much thought is being given to a concern for the individual in the larger context of their society and subgroups within society.[12]

      But even with this reading, it is not the case that BHL is more in line with 19th century thinkers. Their emphasis on the impact of libertarianism on individuals is an influence of the figures they often criticize. While sharing many of their views, BHL figures want to distinguish themselves from the arguments most readily accepted today as developed by Cold War libertarian thought.

      While it goes unstated in the article, an obvious motivation for Z&T is to defend themselves against contemporary critics who want to challenge their break from the Cold War strain of libertarian thought. Yet many attacks on BHL are not motivated by serious philosophical scholarship, but by an antiquated theory of social change that required libertarians to associate with conservatives to defeat the threat of communism. The suggestion that libertarianism can be grounded in philosophical justifications or policy prescriptions more aligned with the left than the right is frightening to those who have grown accustomed to a particular way of viewing the world for nearly half a century.

      BHL’s commitment to the mid-level principles of liberty, not its justifications or policy prescriptions, inducts it into the libertarian tradition. Its scholars do not need to defend their status by driving a wedge between other libertarian thinkers. Just as it is wrong for hard line deontologists to claim that you can only be a libertarian if you take self-ownership as your axiomatic starting point, it is also wrong for certain groups of Objectivists to claim libertarianism is philosophically committed to no underlying justification beyond principles. Claiming a certain justification or set of policies is “more libertarian” than another doesn’t make sense, not because there are no justifications or policy prescriptions for libertarianism, but because libertarianism includes a diversity of strains of thought with competing justifications and policy prescriptions. Libertarianism is a commitment to a set of principles of political philosophy, leaving much room for debate over the justification of those principles and the policy implications of those principles. Individuals outside the libertarian tradition often attempt to essentialize libertarian thought as a monolithic program of justification, principles, and policies. It is much easier for them to (mis)understand and so reject libertarianism that way. However, it is not an accurate description of our school of thought, and it is especially not something that should be encouraged by libertarian thinkers themselves.

      What does this mean for the future of libertarian scholarship? Instead of arguing over who is “more libertarian” based on the justifications they make or the set of policies they advocate, greater value would come from clarifying the principles of human liberty, then analyzing what the most philosophically apt justifications for those principles are, and the best way to apply those principles to the problems facing people in their time. Instead of looking to the past to exclude arguments for the advancement of liberty, we should draw upon the rich intellectual tradition given to us, and from there debate what is “most defensible on philosophic and other grounds.”

      Participating in the libertarian tradition is worthwhile, but it is not an end in itself. To put it in more personal terms: I am a libertarian because I care about liberty, not the reverse.

      Notes

      [1]Z&T do not explicitly state that this is their intention, but it is the apparent goal given their emphasis that BHL “has better claim to [the] title [of traditional libertarianism]” and that “classical liberalism, not axiomatic libertarianism, is the true heir of the liberal tradition.”

      [2]It is not just that the mid-level principles determine the policy prescriptions, though. The justification of principles guides their policy prescriptions with insights to what the principles mean and how they can understand situations.

      [3]Barnett, Randy. “Libertarians and the Iraq War.” Wall Street Journal. July 17, 2007.

      [4]It is easier for libertarians to agree on what the philosophy does not mean.

      [5]Long, Roderick. “In Praise of Bleeding Heart Absolutism.” Cato Unbound. April 4, 2012.

      [6]The anarchist strain of libertarian thought seems to most eagerly embrace this maneuver: a principled opposition to the state means full-on opposition to the state. If you don’t agree that the existence of the state is immoral and unjustifiable, you are a statist, no matter the arguments or positions you proffer. While this may support their criticism of the Rothbardian tradition, this doesn’t fully fit Z&T’s account, as Mises and Rand strongly opposed anarchism.

      [7]Boaz, David. The Libertarian Reader. The Free Press: New York, NY. 1997. pp. xi-xii.

      [8]Locke, John. The Second Treatise on Civil Government. Prometheus Books: Amherst, NY. 1986. p. 20.

      [9]Bastiat, Frédéric. “Property and Law.” As Bastiat put it, the legislators put themselves over the people, and Rousseau put himself over the legislators; see “The Law.”

      [10]Nozick’s Anarchy, State, and Utopia was as much a response to Rothbard’s defense of anarchism as it was to Rawls’ Theory of Justice. Gordon, David. The Essential Rothbard. Ludwig Von Mises Institute: Auburn, AL. 2007. p. 124.

      [11]See commentary by Students For Liberty leaders on feminism and race studies for a sense of what the next generation of libertarians are thinking about. Costa, Moriah. “Why Libertarians Should Embrace Feminism.” Students For Liberty. January 20, 2012. Padilioni, James. “The Law Perverted: A Libertarian Approach to Black History Month.” Students For Liberty. February 1, 2012.

      [12] On this reading, neoclassical libertarians are developing the nonideal theory of libertarianism that supplements the work in ideal theory done by the postwar intellectuals. Nozick’s procedural account of justice argues that the ideal system of justice is based on the legitimate acquisition and transfer of property rights. However, he leaves the theorist wondering how we should best resolve violations of this line of property acquisition or transfer, especially ones that occurred long ago. He posits an ideal world for us to strive for, but with little guidance for how to reach it in our very nonideal world. Nozick, Robert. Anarchy, State, and Utopia. Basic Books: USA. 1968.

      The Conversation

      Some Questions for the Panel

      My first questions are for Zwolinski and Tomasi and have to do with their view of social justice:

      1. You refer to Rawls’ position as one of the “philosophically most sophisticated” formulations of the ideal of social or distributive justice. Do you find his derivation convincing? If so, and if you believe that a libertarian society best satisfies its requirements, would you be in favor of basing your defense of libertarianism on that claim rather than on utilitarian or natural rights arguments?

      2. Why do you find the claims of the “laboring poor” more morally compelling than those of anyone else—including those who are unable to work due to physical handicaps, lack of employment opportunities in their particular environment, or the like? What about the claims of someone who has a large income but even larger medical costs, due to factors not his own fault? What is special about the laboring poor that distinguishes them from other people who are badly off for reasons not their own fault?

      3. There are a large number of laboring poor outside the United States who are a great deal poorer than the laboring poor in the United States. Does it not follow that the effect of policies on the latter group is irrelevant by any non-nationalist version of a “welfare of the worst off” criterion, and that the defense of libertarianism, or any alternative, ought to be based on its effects on people in China and India and Africa, not Watts and Harlem?

      And one small question for Alexander McCobin. You write:

      While libertarians agree on things like the need for minimal government

      Did you really intend, while rejecting any libertarian purity test, to read me, Rothbard, Spooner, and all other libertarian anarchists out of the libertarian movement?

      Alexander McCobin responds:

      I did not mean to exclude anarchists from libertarianism. Note that I said “the need for minimal government” rather than “a minimal government,” as in it is a libertarian value in general to have less government than more government. The purpose was to encompass both minarchism and anarchism.

      Property Absolutism and Social Justice

      We thank Alexander McCobin for reminding us all of the pitfalls of the ideological purity game, in which some in the movement present themselves as “more libertarian than thou.” We agree.

      In our book, therefore, we argue that libertarianism cannot be defined by any rigid set of necessary and sufficient identity conditions. Instead, we think of libertarianism as a family of views. Like any family, the libertarian family has different branches, sometimes tangled and overlapping, yet each with unique characteristics. Here we find a family line the carries the chin of a studious great uncle. There, a branch with the intense eyes of a difficult aunt. We then imagine ourselves being asked to design facial recognition software to pick out the common characteristics—eyebrows and nose, hairline and chin—of this diverse clan. We propose six soft concepts that, when conjoined in any of a great variety of ways, might make our libertarian recognition software go “ping.” So our identificatory system is engineered to be capacious. (We might add that we are skeptical of attempts like Alexander’s to demarcate “real libertarians” by way of a single-sentence definition—but we sincerely do not wish to fall into less-playing-the-ideological-purity-game-then-thou gamesmanship.)

      What’s the Difference (in) Principle?
      Roderick Long isn’t sure just what difference we were trying to draw in our original essay between the libertarianism of Mises, Rand, and Rothbard and their predecessors in the libertarian intellectual tradition. And Roderick helpfully distinguishes between a number of different theses that seem to wind through our view:

      a) whether property rights are based on self-ownership

      b) whether property rights are absolute or nearly so

      c) whether the requirements of justice are exhausted by individual property rights

      d) whether the concept of social justice is to be rejected

      e) whether “the moral justification of free market institutions is logically independent from any claims about the effects of those institutions on the material holdings of the poor”

      f) whether a “significant role for the state in the provision of a social safety net” is to be rejected

      To summarize our response, our project is not at all concerned with (a), nor (as far as we have thought it through) with (c). (d) and (e), however, are quite closely related and are of central importance to our argument, and (b) and (f) are also related though of somewhat secondary importance.[1]

      In short, we think it unfortunate that libertarians such Rand, Rothbard, and Mises are hostile to the idea of social justice. And we regret that, due largely to their tremendous influence, this hostility toward social justice has come to be seen as an essential feature of libertarianism as such. This hostility is philosophically unwarranted and, in a sense we specify, historically at odds with the mainstream of the classical liberal tradition.

      How did this hostility become a defining feature of contemporary libertarianism? Part of the explanation, no doubt, has to do with the real inadequacies of the theories of social justice against which libertarians were reacting. Such theories were very often economically naïve, proposing means inadequate for the ends they hoped to achieve and utterly neglecting the difficult problems of institutional design under nonideal conditions. But, as David Friedman reminds us, theories of social justice face philosophical problems as well. There are a variety of ways of spelling out just what “social justice” amounts to, and many of them will be, in the final analysis, philosophically indefensible. Libertarians are, for example, correct to reject the goal of equality of outcome, not simply for the economic reason that its attempted realization faces severe knowledge problems and generates perverse incentives, but because it is a morally unattractive goal, and one whose pursuit appears to require morally abhorrent means. Most important, until very recently, all theories of social justice were developed from the assumption that little or no moral weight should be assigned to the economic rights and liberties of capitalism.[2] This was a massive moral failing of 20th century theories of social justice. Again, libertarians were quite right to react adversely to that feature.

      However, there is a darker factor that contributes to the antipathy of many contemporary libertarians toward social justice. It is this dark factor within postwar libertarian thought that we find philosophically vulnerable and historically anomalous: a monistic and absolutist focus on property rights as an issue of political morality.

      Property Rights—The One and Only Solution to All Problems of Political Morality?

      Property rights have, of course, always been an important part of the libertarian intellectual tradition. But libertarians more typically think of property rights as one consideration among others relevant to the great moral question of the proper size and scope of the state. Not merely one consideration among others, perhaps, since property rights have often (and rightly) been identified as being especially important moral considerations, both because of their intrinsic nature and their instrumental effects (indeed, in the libertarian facial recognition software we propose in our book, we describe a concern for property as the characteristic nose of the libertarian family—front a center). But property rights were more often conceived of neither in monistic terms as the only relevant moral consideration, nor in absolutist terms as necessarily trumping in all circumstances any potentially competing considerations.[3] Libertarianism prior to the twentieth century was, as we noted in our original essay, a much less systematic political theory. But it was also, and partly in virtue of its lack of systematization, a far more pluralistic moral philosophy—one that embraced a less parsimonious but (in our view) potentially more sophisticated view of the messy terrain of morality.[4]

      For contemporary libertarians, the question of social justice is simple. Any conception of social justice that requires the violation of property rights through redistributive policies is a non-starter. There is no issue of balancing competing moral values, no distinction (often) between more or less morally weighty types of property, no question of the relative urgency of the need to be met compared with the seriousness of the infringement upon property rights involved. Property rights settle all questions, and settle them decisively.

      For earlier libertarians, matters were rarely so simple. Their pluralism did not by any means commit them to social justice, since many of them were writing before that concept even existed (we are in no way claiming that pre-twentieth century libertarians were really crypto-proto-Rawlsians). Rather, our point is that there are elements in their theories that suggest that concern for the poor played a justificatory role in their libertarian theories, and that there is nothing in their theories (such an absolutist commitment to property rights) that is incompatible with a commitment to social justice, properly understood.

      Sometimes, the pluralism of earlier libertarians was explicit. Adam Smith, in a passage quoted by Roderick, defined justice in libertarian-friendly terms as a purely negative virtue—requiring nothing more than leaving other people alone. But Smith did not hold that the enforcement of justice was the only proper role of the state. Smith saw other moral considerations that bear on the question of what states should do, and in at least some cases these other moral considerations trumped the requirements of justice. Consider, for instance, Smith’s endorsement of legal restrictions on the issuance of small notes by private bankers—a practice he saw as susceptible to dangerous fraud and instability:

      To restrain private people, it may be said, from receiving in payment the promissory notes of a banker, for any sum whether great or small, when they themselves are willing to receive them, or to restrain a banker from issuing such notes, when all his neighbours are willing to accept of them, is a manifest violation of that natural liberty which it is the proper business of law not to infringe, but to support. Such regulations may, no doubt, be considered as in some respects a violation of natural liberty. But those exertions of the natural liberty of a few individuals, which might endanger the security of the whole society, are, and ought to be, restrained by the laws of all governments, of the most free as well as of the most despotical. The obligation of building party walls, in order to prevent the communication of fire, is a violation of natural liberty exactly of the same kind with the regulations of the banking trade which are here proposed.”[5]

      But pluralism is not always explicit. Sometimes pluralism can be achieved by incorporating diverse moral considerations into a single consideration like property rights. Consider the case of Robert Nozick. Roderick was puzzled as to why we left him of our “Unholy Trinity” of postwar libertarians. But Jason Sorens has already provided the answer: Nozick, despite appearances, was not a property rights absolutist in the relevant sense. Sorens bases his claim on Nozick’s endorsement of a modified Lockean Proviso. That proviso says that appropriations of previously unowned goods are legitimate only if they do not worsen the position of others (Nozick, p. 178). But, for Nozick, the proviso also includes a “historical shadow” which “excludes his transferring [his legitimately acquired goods] into an agglomeration that does violate the Lockean Proviso and excludes his using it in a way, in coordination with others or independently of them, so as to violate the proviso by making the situation of others worse… (p. 180)” So not only does the proviso forbid one’s appropriating the only watering hole in the desert and charging whatever one wants for the use of it; it forbids charging whatever one wants for the use of a watering hole that becomes the only watering hole in the desert by virtue of all the others drying up.

      Now there’s a sense in which this is still a monistic, absolutist property rights version of libertarianism. Technically, Nozick is not saying that property rights sometimes have to yield in the face of desperate need. Once we know what the property rights are in the watering hole case, we know everything we need to know in order to settle the political morality of the situation. But the monism and absolutism of Nozick’s theory is only skin deep. Desperate need isn’t taken into account as a separate limiting factor on property rights only because it has already been built into the theory of property rights.

      Indeed, Nozick’s incorporation of the Lockean Proviso incorporates a whole host of moral considerations into his theory of property. In explaining why a system that allows the permanent appropriation of unowned resources does not worsen the situation of non-appropriators, for instance, Nozick appeals to the following “familiar social considerations”:

      [Private property] increases the social product by putting means of production in the hands of those who can use them most efficiently (profitably); experimentation is encouraged, because with separate persons controlling resources, there is no one person or small group whom someone with a new idea must convince to try it out; private property enables people to decide on the pattern and types of risks they wish to bear, leading to specialized types of risk bearing; private property protects future persons by leading some to hold back from current consumption for future markets; it provides alternate sources of employment for unpopular persons who don’t have to convince any one person or small group to hire them, and so on. (p. 177)

      This passage is crucial for Nozick’s case for private property and free markets – not because that case rests ultimately on utilitarian considerations, but because Nozick has to appeal to the beneficial consequences of private property to show that the proviso is satisfied. But what this shows isn’t just that Nozick rejects property monism and absolutism. It shows that Nozick is fundamentally committed to something not so far removed from the idea of social justice. The fate of the poor plays an essential justificatory role in his defense of private property. Nozick’s Lockean Proviso insists that private property and free markets are only justifiable if those who fare least well under them don’t fall below a certain baseline of welfare. Now, that’s a far cry from Rawls’ insistence that we maximize the position of the least well-off. And we can quibble about whether the baseline ought to be somewhat higher than how one would fare in the raw state of nature. But once we’ve reached that point, we’ve already determined what kind of theory Nozick’s is. Now we’re just haggling over the price.

      Our concern with Rothbard and Rand is thus not that they are deontologists, whereas we prefer thinkers who are (more) consequentialist such as Milton Friedman and F. A. Hayek. Consequentialism vs. deontology is not the issue. The real issues are monism vs. pluralism, and absolutism vs. non-absolutism.[6] There is nothing objectionable about a position that says that individuals own themselves and should not be coerced, as long as this statement is recognized as a kind of moral rule of thumb that must give way when moral considerations of sufficient weight are brought up against it.[7] Consequentialist moral theories are particularly well-known for being open to this sort of moral cost-benefit analysis. But nothing in the willingness to engage in such analysis commits one to consequentialism, strictly speaking. Pluralism is a garb that consequentialism wears with ease, but that cloth hangs naturally on deontologists and natural rights theorists too.[8]

      A Brief Note on the Matter of Herbert Spencer

      Roderick rebukes us for citing a passage from Spencer that seems to approve of the death of the incompetent, without citing the passage after it that says that “in so far as the severity of this process is mitigated by the spontaneous sympathy of men for each other, it is proper that it should be mitigated.” We are duly chastened. But our point was not to make Spencer look bad.[9] Our point was to show that Spencer tended to say things that made it easy for his critics to make him look bad. And this he did. Perhaps the passage immediately following the one we cited makes Spencer look a little better. But, then again, the passage immediately prior to it makes him look a little worse:

      He on whom his own stupidity, or vice, or idleness, entails loss of life, must, in the generalizations of philosophy, be classed with the victims of weak viscera or malformed limbs. In his case, as in the others, there exists a fatal non-adaptation; and it matters not in the abstract whether it be a moral, an intellectual, or a corporeal one. Beings thus imperfect are nature’s failures, and are recalled by her laws when found to be such.

      A charitable reading of all this can, perhaps, dig up a defensible point. But Spencer seems to be trying to give his readers every excuse he can to not make the effort, and instead to write him off as a heartless monster. And while Roderick is probably right that most people haven’t bothered to read it, Spencer’s follow-up point that if people feel inclined to save the stupid and idle from dying then, well, that’s OK too, probably does little to assuage their concern.

      Social Justice for Libertarians

      So Spencer thinks charity can be a good thing, and we agree. But if that were all he had to fall back on, the case for putting him on the side of the angels would be pretty weak. Thankfully, it isn’t. Spencer’s main point isn’t to deny that the relief of suffering is a proper role of the state because private charity can take care of it. His main point is that the state (and private persons) should allow suffering because in a lot of cases it’s good for people—including the ones who suffer! And this comes pretty close to the kind of public reason story that even advocates of social justice could, in principle, agree with. If a regime that allows individuals to suffer the natural consequences of their bad decisions produces less suffering overall, or less intense suffering by those who suffer most, then it can be endorsed even by one who thinks that the fate of the least well-off plays an important justificatory role in political philosophy.

      Which brings us to social justice. David Friedman trenchantly critiques the maximin decision rule that lies at the heart of John Rawls’s theory of social justice. We think David makes a good point. But social justice survives no matter what one thinks of Rawls’ view.[10] To make conceptual space for a libertarian account of social justice, we take Hayek as our guide.

      Most libertarians think of Hayek as a great critic of social justice. After all, Hayek wrote a book called The Mirage of Social Justice. But less often noticed is that, in that book, Hayek distinguishes two approaches to social justice, one of which he rejects and one of which he affirms. In popular discourse, claims about “social justice” are often invoked to criticize particular patterns of distribution. If the pattern does not match up to some preferred distributional standard, advocates of social justice typically demand state action to redistribute goods so that the morally desired pattern of distribution is achieved.

      Hayek, in our view correctly, rejects this approach to social justice. A more sensible approach sees social justice as a standard for evaluating not particular distributions but institutional systems considered as integrated wholes. So while deriding the former (more common) use of “social justice” as conceptually confused, Hayek affirms the importance of “social justice” in the latter sense: “there unquestionably also exists a genuine problem of justice in connection with the deliberate design of political institutions, the problem to which Professor John Rawls has recently devoted an important book.”[11]

      Social justice, understood this way, may direct us to evaluate social and economic institutions in part by considering what distributional effects they tend to produce.[12] But because justice is a standard that applies holistically to institutional systems rather than to the particular distributions that happen to emerge there, social justice need not call for constant liberty-limiting interventions on the part of government agencies so as to achieve or preserve some preferred pattern of holdings. This point is fundamental to the approach to social justice made famous by Rawls—though, as we explain below, it was actually was pioneered by Hayek. We note with interest that Hayek grasped this deep point while Rawls’ hallmate Nozick apparently did not. (Indeed, again unlike Nozick, Hayek even understood that, on Rawls’ own view, particular interventions to “correct” freely generated distributional patterns to make them conform to some ideal pattern would typically be forbidden by social justice).

      Why does this matter? Well, if social justice is a property of institutional systems rather than of particular distributions, then people who affirm the moral importance of economic liberty have no immediate reason to fear that an affirmation of social justice commits them to advocating (or even allowing) government programs that violate those economic liberties. Instead, like Rawls and Hayek, the affirmation of social justice commits us to ask general questions about rival social systems: namely, considered broadly, and over time, what sorts of social systems tend to help people, to make the best of people’s creative capacities, and to encourage the tide of wealth to rise high, so that even the lowest paid workers do well? Now, the Rawlsians claim that socialist, social democratic, or welfare-statist institutions are best fit to generate that desired state of affairs. How strange if libertarians should hesitate to send out market-society to challenge them on that claim![13]

      But if social justice is a property of institutions rather than distributions, what exactly is social justice?[14] According to Hayek, and much later Rawls, justice is something like fairness. That may seem airy fairy, but Hayek suggests a method by which we might make bring it to ground and then build it up more precisely. In a stunning footnote in The Mirage of Social Justice, Hayek reports that he began thinking of justice as fairness as early as 1940. In London during the bombing, Hayek was facing the problem of deciding which country he should send his children to for safety. This led Hayek to develop a thought experiment that he believed might provide a way for us to model the requirements of social fairness: “we should regard as the most desired order of society the one we would choose if we knew that our initial position in it would be determined purely by chance (such as the fact of our being born into a particular family).”[15] Decades later Rawls would propose a similar device of representation which he called the “original position.” But libertarians should know their family history: Hayek proposed this first. Indeed, we hereby invite our readers to join us in referring forevermore to Rawls’s device as “the (un)original position.”

      Keeping David Friedman’s worries about maximin in mind, we note that there is reasonable debate about precisely which decision rule it would be appropriate to ascribe to the parties in the Hayekian/Rawlsian original position. The key, though, is to track the idea of fairness. And one plausible way (though certainly not the only way) of tracking fairness is to ask Hayek’s question: if you did not know your particular place in society, which type of social order do you think would be most attractive for you to live in? The social order that wins that prize is awarded the crown of social justice.[16] We urge our fellow libertarians to enter the ring and compete for that crown. Again we ask: why not?

      Notes

      [1]We believe in a negative answer to (d) and a negative answer to (e). Indeed, we say no to (d) because of our negative answer to (e). An affirmative answer to (b) has, in our view, led many libertarians to say yes to (d) and (e), whether logic requires them to do so or not (the case that it does seems stronger for (d) than for (e)). And while saying no to (d) and (e) does not logically entail saying no to (f), it does provide some support for it, and saying yes to (d) and (e) would entail saying yes to (f).

      [2]John Tomasi, Free Market Fairness (New York: Princeton University Press, 2012).

      [3]This is, of course, an imperfect generalization. As Roderick’s response essay pointed out, there are certainly some pre-twentieth century libertarians who might plausibly be characterized as property rights absolutists. Lysander Spooner, perhaps, or Auberon Herbert in his later writings. The details of these individuals’ thoughts, and the extent to which they serve as counterexamples to our general thesis, are of course of great interest to us in our larger project. And we are happy to pursue such details further in this ongoing conversation. But so that we might maintain some semblance of brevity and focus, we leave them unexplored in this particular essay.

      [4]What David Friedman said about his own moral view thus seems to us to fit much libertarian thought prior to the twentieth century. “The implications of my moral intuitions are not as tidy as the theories of Rand or Rawls or, for that matter, Bentham. But then, I know of no a priori reason to expect the truth, in moral philosophy or anything else, to always be simple.”

      [5]Adam Smith, An Inquiry into the Nature and Causes of the Wealth of Nations, II.2.94.

      [6]This is why we included Mises in our list. Mises is a consequentialist, but he nevertheless appears to be a kind of property rights absolutist.

      [7]Of course, probably all but the most implausible “absolutist” theories allow their absolute requirements to be overridden in some circumstances. Nozick makes explicit allowance for overriding side-constraints in cases of “catastrophic moral horror.” And Rand argued that it was a mistake to apply the rules of conduct for normal human existence to emergency or “lifeboat” situations. So perhaps absolutism falls somewhere on a continuum.

      [8]On this, we are in complete agreement with David Friedman, who writes that “The version of libertarianism that seems most plausible to me is one where respecting rights is seen as a good thing, a value in itself as well as a means to other values, but not as a value that trumps all others.”

      [9]One of us, in fact, has written in defense of Spencer.

      [10]For one suggestion regarding how libertarians could retain the spirit of Rawlsian social justice while distancing themselves from the implausible details of the particular two principles of justice Rawls settles on to embody that spirit, see Kevin Vallier, “Neo-Rawlsian Libertarianism: Two Principles of Justice for Bleeding Hearts.”

      [11]Mirage, 100.

      [12]For a discussion of this point in the context of Nozick and Hayek’s critiques of social justice, see Matt Zwolinski, “The Libertarian Critique of Distributive Justice.”

      [13]The questions we list here do not identify a unique and precise standard for judging between alternative social systems. Is that social system best which maximizes the primary goods of the least well off (representative) person, as Rawls suggests? If so, what precisely goes on the list of primary goods? And which groups of persons get represented? Do we include paraplegics? Or should we reject the Rawlsian approach altogether in terms of a prioritarian or sufficientarian approach to social justice? We do not propose to answer these questions here. Doing so would require defining and defending a particular conception of social justice. Our purpose, in contrast, is to argue that libertarians should be open to the general concept of social justice—that the justice of social institutions depends, in part, on their distributional effects, and in particular on their effects on the worst-off members of society.

      [14]We are grateful to Luca Gattoni-Celli for recently pressing this question. (Hey there, Luca!).

      [15]Mirage 132. For a fuller account of Hayek’s original “original position, see Tomasi, Free Market Fairness, Chapter 5, “Social Justicitis.”

      [16]A bit more precisely: a socially just social order would secure foundational protection for a set of basic liberties that prominently includes the economic liberties of capitalism, and the institutions that emerge (or are otherwise put in place) there should allow people to interact in ways that benefit all the responsible and cooperative members.

        Why “Social” Justice? More questions for Zwolinski and Tomasi

        You offer a quote from Hayek which you say “affirms the importance of ‘social justice.’” The quote, however, refers to “justice,” not to “social justice,” and in the next sentence, which you do not quote, Hayek explicitly rejects the latter term.

        You describe the “veil of ignorance” argument as being first proposed by Hayek and then, decades later, by Rawls. The passage you cite was published five years after Rawls published A Theory of Justice; your earlier date is the date at which Hayek says he started thinking about the idea, not the date at which he proposed it. It was not Hayek but Harsanyi who proposed the veil of ignorance almost two decades before Rawls. Unlike Rawls, Harsanyi followed the logic of the argument to where it led—judging a society by its average utility.

        You ask “what sorts of social systems tend to help people, to make the best of people’s creative capacities, and to encourage the tide of wealth to rise high, so that even the lowest paid workers do well?” Is the “so that even” clause intended as an essential criterion for a good society or merely an observation about one of the attractive features of a society that maximizes utility?

        In various places, you refer to “concern for the poor.” Do you mean “concern especially for the poor?” For the Rawlsian, the poor occupy a special position. For the utilitarian, the poor count, but only in the same sense that everyone else counts. Those are two quite different positions, and you never clearly distinguish between them. That is particularly disturbing, considering that it is the first which you appear to be advocating but the second that is historically linked to libertarianism.

        What does “social justice,” as you use the term, mean—what does “social” add to “justice?”

        I can see three possible arguments for including social justice within libertarianism. The first is historical—but, while you have offered evidence that classical liberals cared about things other than rights, you have offered no evidence that they were especially concerned about the one thing—the status of the poor—that defines social justice as the term is currently used. The second is philosophical—but you have agreed that Rawls’ derivation is unsatisfactory, and you have offered no alternative.

        The third is to make libertarianism more persuasive to non-libertarians, especially liberal academics. To do that, you need some version of social justice that is both consistent with libertarian principles and sufficiently robust to satisfy liberals. Nozick’s version of the Lockean Proviso is about as close as you are going to come, and I do not think it will do the job. His “baseline” is how well off people would be in a world without private property in land. Would you consider your concerns with social justice satisfied by a world in which the poorest people received an income of a thousand dollars a year, almost everyone else at least a hundred thousand?

        Do you think the people whom you hope to persuade by arguing that libertarians, too, are concerned with social justice would consider that such a world satisfied their concerns?

        On “Concern for the Poor”

        by Matt Zwolinski

        In David Friedman’s first essay, he expressed concern that our original 3000+ word essay omitted a discussion of the connection between classical liberalism and utilitarianism. In his most recent essay, he again defends his utilitarianism against our social justice, as both more historically in line with the classical liberal tradition and as more philosophically defensible on its merits.

        It is, of course, relatively easy to leave things out of a 3,000 word essay on a topic as capacious as the history of classical liberal thought. It is even easy to leave things out of an 80,000 word book on the topic, and we fully expect to do so.

        But our omission was not meant as a slight. Utilitarianism has indeed played a significant role in the development of classical liberalism, though it has typically been a utilitarianism of a distinctively non-Benthamite sort. Not that Bentham did not have his impressively libertarian moments. But it is with good reason that utilitarians like Mill and Spencer believed that human happiness could not be advanced, nor human liberty preserved, without rather strict adherence to rules which themselves made little reference to matters of utility.

        As David notes, utilitarians care about the poor in the same way they care about everyone else: their interests are to be taken into consideration equally along with the interests of everyone else. Advocates of social justice, in contrast, seem to care about the poor in a deeper sort of way: in Rawls’ version, the interests of the least well-off have a very strong moral priority over the interests of everyone else.

        This is a fine and important distinction for philosophers to make. But it’s worth noting that for most of the real world problems that the classical liberals were concerned about, it is a distinction without a difference. Are we concerned with the poor because their interests qua poor have a special non-derivative moral significance? Or are we concerned with the poor because we believe that governments have an obligation to serve the interests of all their citizens, and the poor are the ones who are most likely to suffer when governments fail in this obligation? In both cases, a special sort of concern for the poor is warranted. Philosophers can come up with thought experiments in which these two sources of concern come apart. But in cases ranging from Smith’s discussion of the mercantile system, to Spencer’s discussion of poor laws, to Tucker’s discussion of the money monopoly, they converge. The government policies against which classical liberals inveighed most loudly were ones that were bad on utilitarian grounds, bad on natural rights grounds, but especially bad for the poor.

        Again, we do not wish to claim that the early classical liberals had a fully fleshed out conception of social justice. What we are claiming is that they had a concern for the poor, whether derivative or non-derivative, that played an important justificatory role in their theory, and that would not be incompatible with a fully fleshed out conception of social justice that might be developed today. [1]

        While we’re at it, we should note that our talk of “concern for the poor” has been a bit sloppy as a way of characterizing social justice, both as a philosophically defensible idea and as one with roots in the classical liberal tradition. Those who are concerned with social justice will, it is true, often be concerned about poverty. But on the most plausible conceptions of social justice, and on the conceptions that resonate most strongly with the classical liberal tradition, poverty as such will be neither a necessary nor a sufficient condition for having a valid claim of social justice. Some people will be poor but have no such claim—lazy surfers who are poor simply because they choose not to work, for example. Others will not be poor but will have a legitimate claim of social justice—those, perhaps, whose opportunity to live according to their religious beliefs is unfairly restricted by political institutions. There’s more to social justice than ensuring that people have enough money, and this, in our opinion, is an area in which the classical liberal tradition simultaneously shines and nevertheless still has some important work to do.

        Note

        [1] Such as that developed at length by my co-author, John Tomasi, in Free Market Fairness (Princeton University Press, 2012).

          Best of the Blogs: Where Next?

          A selection of responses to this month’s topic, gathered by the editors.

          “Sticking Rawls into libertarianism is like attaching a washing machine to a soufflé.” In a wide-ranging and sharply worded essay, Todd Seavey casts doubt on the whole enterprise of libertarian social justice. A small excerpt:

          I know how annoying and simpleminded the doctrinaire can sound, especially to professors who treasure nuance, but how can the liberal-tarians dismiss those libertarians who fear de-emphasizing property will quickly yield statism – when the liberal-tarians are living proof that watering down the property rights rule immediately (sometimes in the same sentence!) spawns talk of redistribution and government welfare provision? Have you not stopped to think about this “coincidence”? Am I Charlie Brown that you expect me to try kicking a non-property-centered philosophy even if you keep yanking it away at the last moment and putting some sort of small, ostensibly harmelss welfare state or carbon-trading scheme or something in its place? Do you really think markets work or not?

          And this is why – as a rule-utilitarian (and not a deontologist) – I don’t want people to treat property as just one mushy value amongst other mushy values (parliamentarianism, feminism, whatever). Almost no one adheres to anarcho-capitalist rules for truly arbitrary reasons and says hell-or-high-water, after all. They know and rightly fear the alternatives – as the liberal-tarians have once again demonstrated that they should!

          Update: In another post, Seavey adds:

          If you treasure your status as intellectuals as much as you seem to, there comes a time to admit you’re wrong, and it would be impressive and admirable for the BHL faction to do so immediately after the release of the liberal-tarian manifesto Free Market Fairness. Indeed, they are plainly morally obligated to do so, as, all joking aside, they are attempting to dilute the one philosophy that can save this society by transmuting it into the very philosophy that is rapidly destroying society, on campus and in Washington, DC.

          There is not some aspect of this that their opponents “don’t get,” “need to study more,” or are “resisting.” BHL is false and destructive, and, as usual, I have been entirely too kind in my criticisms. I will not continue to be if they persist in this self-indulgent, socially destructive, historically-ignorant con game. What they are doing is, in a word, evil. I think they must know it on some level, but they pride themselves on playing this particular philosophical game.

          How Rawlsean Is It, Really? Kevin Vallier suggests that we don’t have to be very Rawlsean at all — the washing machine and the soufflé can perhaps sit comfortably across the room from each other:

          I don’t think the neo-Rawlsian view (to the extent that there is one) has to draw that deeply on Rawls. In fact, from what I can tell, even John [Tomasi], who is more Rawlsian than any of us, rejects maximin. So my goal is to sketch a broadly Rawlsian theory that fits within what John calls the market democratic research program, a program that develops theories that combine a commitment to thick economic liberty with a concern for the least-advantaged….

          On the libertarian view, there is no presumption in favor of an equal distribution of goods because there is no presumption in favor of any goods existing at all. Goods must first be produced before they are distributed and social institutions should be arranged so that this basket of goods is as large as it can be. Then the least advantaged only have a claim on the cooperative surplus if it is required to ensure that their lives meet a decent threshold. Otherwise goods should be distributed in accord with the Liberty Principle. For this reason, I think the neo-Rawlsian principle of social justice is sufficientarian. The sufficiency threshold is bound to be controversial. But we do not need to set the threshold to proceed.

          Lumping or Splitting? At Coordination Problem, Peter Boettke writes:

          [I]t seems that the all too common lumping of Mises with Rothbard and Rand is something that should be challenged.

          So I want to suggest that the syllogism Mises = Austrian economics; Mises = Rothbard; Rothbard = Austro-libertarianism; demonstrate fundamental flaws in Austro-libertarianism; therefore, demonstrated that Rothbard is flawed, therefore, Mises is flawed; is in fact what is flawed and must by rejected serious thinkers in social philosophy and political economy.

          Mises may have problems, but they are not the ones supposedly identified in Rothbard and Rand. Mises is a rule utilitarian, and the critical idea in his social theory is the theory of social cooperation under the division of labor.

          Libertarianism Isn’t Capitalism Or so writes Jason Sorens:

          I would stress that libertarianism and capitalism are two separate concepts. One may endorse libertarianism without endorsing capitalism and vice versa. Or one might hold, as I do, that the moral justification of libertarianism is largely independent of welfarist considerations, but that the moral justification of capitalism is largely dependent on welfarist considerations. Zwolinski and Tomasi never draw that distinction, but I think it is an important one…

          It is certainly true that whether the Difference Principle justifies ongoing income redistribution is a matter of contingent fact. Nevertheless, it’s pretty implausible that a pure free market, even with abundant private charity and mutual aid, could satisfy the Difference Principle. Remember that Rawls’ theory holds that just institutions must maximize the position of the representative least advantaged person. That is, when analyzing institutions comparatively and normatively, we must select that order that best guarantees the welfare of the worst-off.

          Compare a pure free market with abundant mutual aid and private charity to that same society with a 1% income tax on annual personal income above $5 million, combined with an earned income tax credit of, say, $500 per year (those numbers seem roughly right for balanced budgets). Which society best secures the welfare of the worst-off: the first or the second? The deadweight loss from the tax-and-transfer system of the second society would be minuscule, and it is wholly implausible that this loss would outweigh, even in the long run, the $500 benefit that the poorest would receive.

          Herbert Spencer’s Revisions Mike Konczal comments on the discussion; the context is a post discussing among many other things the revisions Herbert Spencer made over the years to his landmark work Social Statics.

          Konczal argues that Spencer watered down his radicalism over the years, and I agree. The Spencer of 1851 favored communal land ownership — tripped up, I would argue, by what Robert Nozick would later term Lockean Proviso. John Locke stipulated that it was only permissible to acquire land in the state of nature when enough was still left for a similar acquisition by others, thus casting a shadow (perhaps inadvertently) on all land acquisition ever since.

          This was a difficulty that would not in my view be untangled until Nozick did so in Anarchy State, and Utopia, where he pointed out that the institution of land ownership itself generates wealth, even, in the long term, for those who never own any. Because they profit from exchange, they have not really been wronged. (An alternate solution, of course, comes from geolibertarian theorist Henry George.)

          But anyway. Konczal writes:

          I’m currently reading Free Market Fairness by John Tomasi; here they are debating the theory at Cato Unbound. In order to explain the difference between classical liberals and high liberals like Rawls, Tomasi uses the thought experiment of how each would set the rules for a game of Monopoly. He does it in the book and he does it in this blog post. Classical liberals want equal rules for everyone in Monopoly and are okay with unequal starting money; High Liberals are concerned about inequalities in wealth, even after the game begins. And so on.

          How would Herbert Spencer approach Monopoly, a game entirely about private property in land? The crucial part of the game isn’t the issue of fairness between players on turn one, it’s how fair the game is to people who start on turn 30. At that point, the productive capital (all the land cards) have been purchased. The turn 30 players would run around the board hoping not to be bankrupted. Maybe they’d pass go enough times to build enough wealth to buy some land, but most likely all their income would be siphoned off in rents by the turn one players…

          So if you asked the young Herbert Spencer how to fairly set up the rules of Monopoly, he’d probably say “there are no fair rules to this game,” flip the board over and walk away. Like a boss.

          Class Analysis Brian Doherty offers a long, detailed summary and response at Reason. It’s hard to pick out just one passage to comment on, but this one is among my favorites:

          [Roderick] Long ends with his belief that when you add class analysis and exploitation into the mix, you can get a libertarianism that is both concerned with social justice (making sure everyone gets what is rightfully theirs) and absolutist on property rights.

          I’ve always enjoyed this argument from contemporary left-libertarians, and while doubtless many income inequalities in the modern world are caused or exacerbated by state-propped class crime and warfare, I don’t think it is proven that all the income inequalities that bug a Rawlsian or a modern progressive of less refined philosophical beliefs actually will disappear in a freed market. It’s a great thing to make progs think about–how much of what you hate about modern income and power inequality is actually the state’s fault??–but I don’t think the answer is, all of it!

          By class analysis, we’re not talking about who owns the means of production. We’re talking about who owns the means of coercion. This is both the original form of class analysis and also one that libertarians who value private property can consistently embrace, as Murray Rothbard among many others have done.

          It’s All About the Utility Mike Rappaport has a three part series on how he came to think of himself as a bleeding heart libertarian:

          Somewhere along the line – I think it was in the early 1980s – I became convinced of a type of utilitarianism, namely welfare consequentialism. I had been a Nozickian libertarian but became persuaded of consequentialism. Much of the responsibility, I believe, must be assigned to Richard Epstein and Friedrich Hayek (even though Hayek claimed not to be a utilitarian). It was easy, as a libertarian, to become a welfare consequentialist. If libertarian institutions have the good effects that libertarians believe they do, then welfare consequentialism provides a strong basis for libertarianism.

          The Bleeding-Heart Absolutist Strikes Back

          Let me start eirenically by noting what we all seem to agree on: that consequences (including consequences for the poor) matter for the justification of libertarianism; that they are not all that matters, since non-consequentialist considerations are also relevant; and that libertarianism of some sort meets both consequentialist and non-consequentialist criteria pretty well.

          Among the points where I think we disagree are the relative weight to be put on consequentialist versus non-consequentialist considerations (here David is, I think, more consequentialist than Matt and John are, while they in turn are more consequentialist than I am; I’m not sure where Alexander stands on this), the extent to which the former are conceptually independent of the latter (much less for me than for my fellow symposiasts, as far as I can tell), and the theoretic level(s) at which consequentialist considerations should most come into play (fairly high, for me). But there are still areas where I need clarification.

          Absoluteness, Monism, and Trumps

          Matt and John counsel against treating property rights “in absolutist terms as necessarily trumping in all circumstances any potentially competing considerations,” and express agreement with David’s preference for a form of libertarianism in which “respecting rights is seen as a good thing, a value in itself as well as a means to other values, but not as a value that trumps all others.”

          I’m a bit puzzled by this. Ordinarily, to call something a right (at least a full-fledged as opposed to a prima facie right) just is to say, inter alia, that it trumps other considerations; in Ronald Dworkin’s memorable phrase, rights are trumps – and so claims that do not trump are not rights. To say that S has a right to be treated in manner M is to say that a) others ought to treat S in manner M, and b) it is permissible for S, or S’s agent, to compel said others to do so. So once it’s agreed that S has the right, there’s no further question as to whether the right ought to be respected; or, to put the matter the other way around, once it’s agreed that a claim should be overridden, such a claim is no longer regarded as a right (again, except prima facie). (Admittedly the matter is a bit more complicated; if the permissibility of compelling compliance is qualified or delayed owing to, say, proportionality requirements, yet not eliminated, we might still want to say that the right stands.)

          On a closely related issue, although I used the term “absolute” (and its cousins “absolutism” and “absolutist”) in my previous post, I did so with trepidation, for it’s a term I find confusing and usually try to avoid (except where it’s just the negation of relativism). For one thing, it’s often unclear whether it’s supposed to be a comparative term or not. Are some rights absolute and all others non-absolute, making absoluteness all or nothing, or does absoluteness come in degrees? For another, it’s unclear how to distinguish the case of rights’ differing with regard to absoluteness from the case of rights’ differing with regard to their objects.

          Suppose that according to theory A I’m entitled to hit anything I choose, while according to theory B I’m entitled to hit anything I choose except other people. Should we say that theory A grants me an absolute, and theory B a non-absolute, right of hitting? Or that theory A grants me a more absolute right than theory B? Or should we simply say that here we have two equally absolute rights, but one is an absolute right to hit anything, and the other is an absolute right to hit non-people? Once we’ve determined the object of the right – what it is a right to do – what more is there to say about its absoluteness or lack thereof?

          In addition to the issue of absolutism, Matt and John now introduce the monism/pluralism distinction; but this runs the danger of being equally obscure, since it’s not clear whether it applies to the content of principles or to their justification. Can one have a pluralistic justification of a monistic principle?

          Perhaps so. In explaining why Nozick isn’t included in the “Unholy Trinity” of Mises, Rand, and Rothbard, Matt and John note that for Nozick considerations of need, and indeed a “whole host of moral considerations,” fail to be a “separate limiting factor on property rights” simply because they have “already been built into the theory of property rights.” Well, sure; but how is that any less true for Mises, Rand, and Rothbard? For Rand and Rothbard, principles of justice rest on the conditions required for each human being to flourish as a rational agent, which seems a fairly pluralistic basis. (In particular, for anyone who is tempted to interpret Rand’s theory as monistic, I recommend Chris Sciabarra’s analysis of Rand as a dialectical, multidimensionally integrative thinker in Ayn Rand: The Russian Radical.) And for Mises, the justification of libertarianism is its giving the widest scope for everybody to achieve whatever ends they may happen to have. If that’s not pluralism, what could be?

          The closest thing to a monistic libertarian theorist that I can come up with would be someone like Hans-Hermann Hoppe, who bases self-ownership on the purported impossibility of rejecting it without self-contradiction – an argument which, if it worked,[1] would make property rights independent of any other value. But this approach is hardly typical of most libertarian thinkers; and even Hoppe makes much of the fact that respect for property rights would bring about various social outcomes he regards as valuable, even if this fact is not part of his “official” justification for such rights.[2]

          More broadly, there are a variety of ways in which libertarianism can be pluralistic. There are values with which it might be aligned because these values are part of its justification; or because these values follow from values that are part of its justification; or because these values are necessary for its implementation; or because these values are made (more) possible by its implementation; or because these values are necessary for the proper application of its principles.[3] Libertarianism can be social-justice-oriented in all these ways too; we might call a version of libertarianism social-justice-oriented if concern for the poor is part of the justification for its principles; or if it highlights the ways in which applying those principles (however justified) addresses that concern; or if it appeals to that concern in determining how to apply its principles; or if it shows that concern to be worthy of our attention for the same reason that libertarian principles are, and so forth. Note that none of these ways necessarily requires thinking of libertarian rights as overridable (whether easily or at all) by social-justice concerns.

          Matt and John also say that they find “nothing objectionable” in the view that “individuals own themselves and should not be coerced, as long as this statement is recognized as a kind of moral rule of thumb that must give way when moral considerations of sufficient weight are brought up against it.”

          But first, calling this a “rule of thumb” seems to go beyond saying merely that exceptions may be called for in extreme circumstances; the language naturally suggests that self-ownership may be fairly easily and casually overridden. I don’t think Matt and John intended this reading, but their wording invites it.

          Moreover, it seems like a violation of human dignity to treat self-ownership as something easily overridden; and since, for those who accept self-ownership, property rights must either be grounded in self-ownership or rejected entirely,[4] that means property rights will have to be fairly robust. Nozick’s (well, Rand’s originally) demonstration that taxation is on a par with forced labour is not something that bleeding-heart libertarians, of all people, should be losing sight of; compassion should be concerned with more than distributional effects alone. (Yet that doesn’t mean that libertarianism must be insensitive to distributional effects, since such considerations can (and typically do) enter into the justification of self-ownership in the first place.)

          I incidentally welcome Alexander’s distinctions among justification, principles, and policy. (I’m not inclined, though, to agree with his inference that these distinctions render pointless the debate over whether one form of libertarianism is more thoroughgoingly libertarian than another. After all, any given theory is more than just a collection of its author’s opinions; it’s a structure in the space of reasons, with its own internal dynamic, its natural tendencies of development, its stresses and strains, its “objective tendency of the problematic.”) One upshot of these distinctions is that consequence-sensitivity and social-justice-sensitivity can come in at different levels. Suppose theorist A uses entirely or almost entirely deontological considerations to justify her principles, but then allows the principles in their application to be fairly easily overridden by concern for consequences (whether for the poor or for everyone), while theorist B allows such consequentialist considerations (whether largely or entirely) to shape her justification of her principles, but once her principles are in place, she permits few if any exceptions to them. Is theorist B more “absolutist” than theorist A? Maybe. But is one of these theorists more consequence-sensitive and/or more social-justice-sensitive than the other? That’s far from obvious.

          Another question for Matt and John: suppose that a given libertarian denies that e) “the moral justification of free market institutions is logically independent from any claims about the effects of those institutions on the material holdings of the poor,” but affirms that d) the concept of social justice is to be rejected. (Hayek might conceivably fit that description.) Would such a libertarian count as social-justice-oriented? If so (perhaps on the grounds that substance trumps terminology), then the extent to which prominent 20th-century libertarians were genuinely anti-social-justice may need to be reconsidered.

          I continue to think, though, that its focus on merely distributional concerns renders the Rawlsian version of social justice rather tepid, leaving out such issues as exploitation, expropriation, class rule, and social control – to say nothing of patriarchy and white supremacy – that have been crucial not only to the radical left (as conventionally conceived) but also to 19th-century libertarians and their 21st-century admirers. (One way to describe the difference between the branch of bleeding-heart libertarianism to which Matt and John belong and the branch to which I belong is to say that they’re trying to show that libertarianism can satisfy Rawlsians while we’re trying to show that libertarianism can satisfy Chomskyans.)

          Utilitarianism versus Social Justice and Natural Rights


          David wonders why social justice, particularly in its Rawlsian form, should be thought preferable to utilitarianism. One reason, I think, is that the Rawlsian concern with mutual advantage rather than overall advantage forbids sacrificing the few to the many, something utilitarianism allows in principle even though it generally counsels against it in practice. And allowing it even in principle, and thus treating a harm to A as something that could conceivably be made up for by a benefit to B, ignores the separateness of persons – in ways that both Rawls and Nozick identify.

          David objects that the mutual-advantage approach implausibly forbids even slight reductions of one person’s welfare for the sake of others; but this is so only if one takes welfare as an independent variable, which strikes me as a mistake – one of the many reasons I’m an Aristotelean rather than a Rawlsian. If the relation between justice and benefit is a two-way street, so that considerations of justice can play a role in shaping what counts as a benefit instead of solely vice versa,[5] then David’s counterexample becomes impossible.

          There are other problems with the Rawlsian approach, of course, including the arbitrariness of the baseline; after all, the Pareto criterion and the Rawlsian difference principle both favour mutual rather than overall advantage, but disagree over what to take as the baseline, Pareto privileging actuality and Rawls privileging equality – both dubious baselines for historical-entitlement reasons that Nozick explained well. (Hayek’s version of the veil of ignorance seems vulnerable to similar objections.) But the focus on mutual advantage at least allows Rawlsian considerations some claim to play a role in the justification process, whereas utilitarianism lacks any such status.

          David prefers utilitarianism not only over Rawlsian social justice but also over traditional self-ownership theory; one reason for this preference is that “the version of libertarianism exemplified by Rand, Rothbard, and their followers” lacks a “logical foundation sufficient to persuade the unbeliever of its strong claims.”

          But isn’t this just as much a problem for the utilitarian as for Rand and Rothbard (or Rawls, or any of us)? It’s a mistake to think that by appealing to utilitarian considerations one avoids reliance on controversial value-judgments. After all, utilitarianism is a controversial theory too; even David himself doesn’t believe it![6]

          Another of David’s criticisms of self-ownership theory is the purported absurdity of applying it consistently:

          I cannot turn on the lights in my house without prior permission from every landowner whose ability to see them demonstrates that my photons are trespassing on his property, and if I fall out my apartment window and end up clinging to a projecting flagpole a floor lower, I must let go and fall to my death when ordered to do so by its owner.

          But one need not turn to utilitarianism to solve such problems; one may merely note, first, that forms of physical interaction that are part of the background conditions of the very existence of human society cannot fall within the scope of actions to be prohibited by its laws, and second, that purely deontological issues of proportionality forbid removing involuntary trespassers from one’s flagpole in such a way as to kill them. (Given Rand’s dialectical insistence on context-keeping, it’s particularly odd to saddle her with these implications. To be sure, there are versions of self-ownership theory that might be vulnerable to these examples – namely versions, such as Hoppe’s, that make the justification of self-ownership completely independent of all other moral values – but most forms of self-ownership theory do not follow that lonely alpine path.)

          Another Note on Spencer


          It’s certainly true that Spencer said many things that make him sound like a heartless bastard. But he also said many things that make him sound like a bleeding-heart lefty, such as his condemnation of wage labour as a form of slavery, and endorsement of workers’ cooperatives as a way of freeing labourers from the tyranny of employers[7] – or this from Social Statics:

          It is very easy for you, O respectable citizen, seated in your easy chair, with your feet on the fender, to hold forth on the misconduct of the people – very easy for you to censure their extravagant and vicious habits …. It is no honor to you that you do not spend your savings in sensual gratification; you have pleasures enough without. But what would you do if placed in the position of the laborer? How would these virtues of yours stand the wear and tear of poverty? Where would your prudence and self-denial be if you were deprived of all the hopes that now stimulate you …? Let us see you tied to an irksome employment from dawn till dusk; fed on meager food, and scarcely enough of that …. Suppose your savings had to be made, not, as now, out of surplus income, but out of wages already insufficient for necessaries; and then consider whether to be provident would be as easy as you at present find it. Conceive yourself one of a despised class contemptuously termed “the great unwashed”; stigmatized as brutish, stolid, vicious … and then say whether the desire to be respectable would be as practically operative on you as now. … How offensive it is to hear some pert, self-approving personage, who thanks God that he is not as other men are, passing harsh sentence on his poor, hard-worked, heavily burdened fellow countrymen ….[8]

          Scrooge-sounding statements and bleeding-heart-sounding statements both permeate Spencer’s writings, often in quick succession; that’s one of the oddities of reading him. But the fact that it is nearly always the Scroogey statements that get quoted, and hardly ever the bleeding-heart ones, suggests to me that it is the readers themselves, and not Spencer alone, who bear the responsibility for finding “every excuse … not make the effort” at interpretive charity.

          As for Spencer’s views on charity in the literal sense, I think Matt and John underestimate the moral importance Spencer places on helping the unfortunate when they summarise him as holding that “if people feel inclined to save the stupid and idle from dying then, well, that’s OK too” – as though Spencer thinks it is a morally neutral matter whether such assistance is offered. On the contrary, Spencer writes:

          [T]he highest form of life, individual and social, is not achievable under a reign of justice only; but … there must be joined with it a reign of beneficence. … A society is conceivable formed of men leading perfectly inoffensive lives, scrupulously fulfilling their contracts, and efficiently rearing their offspring, who yet, yielding to one another no advantages beyond those agreed upon, fall short of that highest degree of life which the gratuitous rendering of services makes possible. Daily experiences prove that every one would suffer many evils and lose many goods, did none give him unpaid assistance. The life of each would be more or less damaged had he to meet all contingencies single-handed. Further if no one did for his fellows anything more than was required by strict performance of contract, private interests would suffer from the absence of attention to public interests. The limit of evolution of conduct is consequently not reached, until, beyond avoidance of direct and indirect injuries to others, there are spontaneous efforts to further the welfare of others. …

          We have seen that cooperation and the benefits which it brings to each and all, become high in proportion as the altruistic, that is the sympathetic, interests extend. The actions prompted by fellow feeling are thus to be counted among those demanded by social conditions. They are actions which maintenance and further development of social organization tend ever to increase; and therefore actions with which there will be joined an increasing pleasure. From the laws of life it must be concluded that unceasing social discipline will so mold human nature, that eventually sympathetic pleasures will be spontaneously pursued to the fullest extent advantageous to each and all. [9]

          Moreover, not only does Spencer hold that “in so far as the severity of this process [of natural selection] is mitigated by the spontaneous sympathy of men for each other, it is proper that it should be mitigated,” but he goes on to add that although “there is unquestionably harm done when sympathy is shown, without any regard to ultimate results,” even in that case “the drawbacks hence arising are nothing like commensurate with the benefits otherwise conferred.”[10] In other words, not only is prudent charity better than no charity, but imprudent charity is also better than no charity.

          I would also want to resist Matt and John’s suggestion that the “main point” of Spencer’s case against government welfare is that the state “should allow suffering because in a lot of cases it’s good for people – including the ones who suffer.” Spencer’s “main point” in opposition to government welfare is that forcible redistribution violates the Law of Equal Freedom. And Spencer’s case for the Law of Equal Freedom is that “man’s happiness can be obtained only by the exercise of his faculties,” and “to exercise his faculties he must have liberty to do all that his faculties naturally impel him to do” – a derivation that makes no reference to salutary suffering.[11]

          Finally, I think Matt and John are mistaken to seize on Spencer as an alternative to the property-absolutism of 20th-century libertarianism; for Spencer surely has to count as (what they call) a property absolutist too. Spencer shows this most clearly, ironically enough, in his arguments for rejecting private property in land: if property rights were applied to land, Spencer tells us, such rights would have to be unlimited, as all property rights are – and in the case of land this would lead to absurd consequences. As Spencer sees it, the choice is between an unlimited right and no right; and those who would plead instead for a limited right to land he derides for thinking that “Truth must always be spiced with a little Error,” and insists instead that any principle governing property must be “pure, definite, entire, and unlimited.”[12] Hence for Spencer we get no private property in land, and unrestricted private property in everything else.

          Notes

          [1] For what it’s worth, I don’t think it does; see my “The Hoppriori Argument.”

          [2] I have serious disagreements with Hoppe as to which social outcomes are valuable; happily, these are also areas where I disagree with him as to which outcomes are likely results of respect for property rights.

          [3] For some of the relevant distinctions here, see Charles Johnson, “Libertarianism Through Thick and Thin.”

          [4] For an argument to this effect, see my “The Paradox of Property.”

          [5] For details see my “Eudaimonist Libertarianism” and “Twelve Theses on Libertarian Eudaimonism.

          [6] David invites us to consult the index to his book The Machinery of Freedom for his criticisms of utilitarianism – but directs us, alas, to a PDF copy that does not include the index.

          [7] Principles of Sociology VIII.20.

          [8] Social Statics, Chapter XX.

          [9] Principles of Ethics I.8, 14; V.1.

          [10] Social Statics, Chapter XXVIII.

          [11] Social Statics, Chapter IV.

          [12] Social Statics, Chapter IX.

          Defining Libertarianism

          While I have not yet read Z&T’s developed account of what constitutes the libertarian family, I think Z&T’s conception of libertarianism as a family of ideas/figures is interesting, and at a basic level true, but from their description,[1] I want to push back on using this framework to demarcate (or not demarcate) libertarianism. A family is defined exclusively by the people within it and who the family chooses to welcome into it.[2] The libertarian tradition, though, is not simply the people we welcome into the scholarship. There must be some strain of intellectual unity to the tradition that brings everyone together. This is where principles come in. By recognizing the libertarian tradition as a commitment to a common set of principles, we can admit the diversity of justifications and policy prescriptions to the libertarian tradition, but also identify an intellectual commonality among libertarians that separates our school of thought from others in a sounder manner than “we say so,” or being invited to the same conferences, receptions, and book releases as other libertarians.

          To clarify my point in response to Roderick Long,[3] I don’t mean to suggest we cannot say that a philosophy is more or less libertarian than another. However, I believe the standard to employ in doing so takes place at the level of principles. To say someone is not libertarian is to claim they are not committed to libertarian principles, whether explicitly or implicitly.[4] To say someone is more libertarian than another is to say their principles are more aligned with libertarianism. However, I will add two important caveats to this: First, I am sympathetic to Z&T’s worry about defining libertarianism in a sentence. I pointed out in my first post that even identifying the principles that constitute libertarianism is difficult and is sure to rouse debate amongst libertarians. I am not about to offer the gold-standard philosophical definition of libertarianism, and would be skeptical of anyone who said they were. But that doesn’t mean this is not the right place to look for what differentiates libertarianism or make progress on clarifying those principles. Second, debates over whether certain justifications or policies are more libertarian than others may be possible if it could be shown that it was logically necessary to hold justification X to achieve libertarian principles (or a particular libertarian principle), or that policy Y was the logical application of libertarian principles.

          However, I think this is a very difficult thing to do. An easier task would be to show that libertarian principles logically preclude certain policy prescriptions, for example that libertarian principles logically preclude the adoption of a communist state. Or, one could also argue that a certain justification could not lead to libertarian principles.[5] But I would expect these efforts to, at best, exclude extreme views rather than offering substantial contributions.

          What Morally Justifies an Economic System?

          Based on Z&T’s first article it seems as though there are five potential criteria for moral justification of an economic system other than one based strictly on deontic rights protection:

          1. The system delivers the greatest payout to most well-off in society.
          2. The system delivers the greatest payout to the average members in society
          3. The system delivers the greatest payout to the least well-off in society
          4. The system delivers the greatest payout to everyone in society (i.e. including the least, average, and most well-off)
          5. The system is possible

          While I do not think there is anyone who endorses #1 in today’s world, Z&T’s first article seems as though it was written to refute that standard. David Friedman seems to be advocating #2 with his defense of utilitarianism.[6] Using this demarcation, I find Roderick Long’s point (e), “the moral justification of free market institutions is logically independent from any claims about the effects of those institutions on the material holdings of the poor,” to be an odd position. As written, none of the authors Z&T use as foils would embrace (e), since it would seem to require them to embrace either #1 or #2, which none do.

          If what Roderick had meant, and what Z&T want to defend, is that “the moral justification of free market institutions is logically dependent on the effects of those institutions on the material holdings of the poor,” then we would have an intellectually intriguing position to debate. This version contains a two-step analysis: First, that the standard for moral justification of an economic system is its effect on the material holdings of the poor. Second, that the free market does better on this standard than others and so is the most moral system. I think Roderick, David, and I would agree to the latter point. The first is the real question at hand: should our standard of justification for an economic system be the advancement of the welfare of those least well-off (i.e. #3)?

          At times, Z&T seem to want #3 and #4 to mean the same thing, but they are separate standards.[7] #4 argues that we should be concerned with the impact of an economic system on everyone without claiming one type of person deserves greater moral consideration. #3 argues that we should grant greater moral weight to the interests of the poor than to others. I am interested in asking Z&T to clarify which of these standards they think is most appropriate for evaluating an economic system. Do they want to argue that we must give greater moral weight the least well-off or do they want to argue that we should treat everyone with equal moral weight, which includes the least-well off?

          Ludwig von Mises doesn’t seem to be appropriately categorized as either an advocate of social justice or in the same camp of absolute property rights advocates. Rather, I think it’s most accurate to claim he employs standard #5. Z&T quote from Liberalism on the question of providing for the least well off, but in the sections surrounding this quote, Mises is explicitly arguing for the importance of having an economic order in general. He argues that there can be no social organization or economic progress without a defense of property rights and liberalism. Yes, the poor will benefit in this system, as will everyone else, but the standard he is employing is simply whether that order can exist or not. He defends liberalism on the grounds of defending the structure itself. And by defending the structure, the poor benefit.

          As Z&T recognize, Rothbard and Rand both also invoke the beneficial impacts of the free market on the least well off in society in their justifications for the system. They certainly do not embrace #3 as the standard for moral justification, but they also certainly reject #1 and #2. For different reasons (as I intend to explain for Rand below), they seem to embrace a version of #4 where everyone benefits from rights protections as a standard for moral justification.

          Was Rand a Property Rights Absolutist?

          Z&T claim that “For contemporary libertarians, the question of social justice is simple. Any conception of social justice that requires the violation of property rights through redistributive policies is a non-starter. There is no issue of balancing competing moral values, no distinction (often) between more or less morally weighty types of property, no question of the relative urgency of the need to be met compared with the seriousness of the infringement upon property rights involved.” They then comment in their response that they did not name Robert Nozick as an opponent of theirs because he was not a property rights absolutist. But if that’s the case, then, as Roderick Long briefly mentions,[8] Ayn Rand should not be a target of their criticism either. I want to develop this point more thoroughly here because Z&T seem to be responding to the strawman version of Objectivism.

          The right to property is not primary in Objectivism. The right to life is. The right to property is dependent upon a legitimate claim to the right to life and can be overridden by claims of the right to life. For example, a standard right to property is overridden in cases of emergency where standard forms of interaction do not apply. If someone becomes shipwrecked and manages to swim to an island only to find a chain link fence all around it with the island’s owner saying, “you can’t come in here,” there is no violation of rights when the shipwrecked scales the fence and climbs safely to land.[9] The right to life is the origin of all other human rights.

          Property rights depend upon a standard, predictable system of interaction with others. When that system is in place, property rights ought to be protected. But when that system is not in place, those rights do not exist as they normally would. The same could be said of the watering hole in the desert if an emergency situation existed.

          The point of pushback that Objectivism would offer is that we are rarely in these emergency scenarios. A finite supply of watering holes is not the same thing as there only being one watering hole. A limited number of islands is not the same thing as there only being one island. A stagnant economy does not mean people have a right to a job. In a standard world, standard rights protections apply. If things get bad, but we are still living standard lives, rights protects apply. Only when we are in a truly emergency situation do normal rights protections not apply. And these emergencies don’t happen often.

          Now this limitation to the force of property rights is not grounded in an abstract, and so more flexible, concept of “harm.” It may not lead to the rejection of property rights claims as frequently as the Lockean Proviso’s limitation. And it is not principally about a concern for the poor qua poor. It is a concern for those who are in non-standard situations, which preclude normal derivations of the right to life. But it is a limitation on property rights.[10] And that exempts Rand from the main thrust of Z&T’s criticism of property rights absolutism.

          On Anarchism

          Finally, in response to David’s question of whether I was excluding anarchism from libertarianism:[11] I did not mean to exclude anarchists from libertarian thought. Note that I said “the need for minimal government” rather than “a minimal government,” as in it is a libertarian value in general to have less government than more government. The purpose was to encompass both minarchism and anarchism. However, what I am challenging is the notion that anarchism is the logical outcome of libertarianism or the idea that the “truest” libertarians are anarchists, which as I mentioned in a footnote last time, I think is an argumentative strategy employed by many anarchists. For example, I recently saw a Facebook question amongst a group of my friends that asked, “How far do you take libertarianism?” The options provided listed four types of anarchism and minarchism. The question itself suggests that the logical conclusion of libertarianism is anarchism, and the only thing stopping people from being anarchist is a willingness to compromise their libertarianism.[12] But if we treat libertarianism as a commitment to a set of principles rather than set of policies, the question itself does not make sense. Whether one is minarchist or anarchist is not a consequence of “how libertarian” they are, but their endorsement of libertarian principles.[13] And I believe that is a great advantage of my interpretation, both as a descriptive and normative assessment of the libertarian tradition.

          Notes

          [1] Matt Zwolinski and John Tomasi, “Property Absolutism and Social Justice,” Cato Unbound, April 16, 2012.

          [2] Some may argue that families are determined by blood relation, but non-blood relatives can enter the family whenever we embrace them as one of the family. The standard for familial relation is solely the recognition of someone as part of the family by some part of those who are already in the family (what part or how much is not something appropriate to explore here).

          [3] Roderick Long, “The Bleeding Heart Absolutist Strikes Back,” Cato Unbound, April 23, 2012.

          [4] Frédéric Bastiat makes this point most strongly in The Law when he recounts: “Mr. de Lamartine once wrote to me thusly: ‘Your doctrine is only the half of my program. You have stopped at liberty; I go on to fraternity.’ I answered him: ‘The second half of your program will destroy the first.’” In considering what principles we are committed to, we must be careful to examine which principles cohere and which contradict.

          [5] Which is the strategy typically employed by opponents of libertarianism: “If you care about Z, you cannot be a libertarian.” One thing I greatly admire about Bleeding Heart Libertarianism is that it is doing a strong job refuting this argument where Z is “the least well-off in society.”

          [6] David Friedman, “Why ‘Social’ Justice? More Questions for Zwolinski and Tomasi,” Cato Unbound, April 17, 2012.

          [7] This point is similar to and builds upon David’s distinction between “concern for” and “concern especially for” the poor, in ibid.

          [8] Roderick Long, “The Bleeding Heart Absolutist Strikes Back,” Cato Unbound, April 23, 2012.

          [9] Ayn Rand, “The Ethics of Emergencies,” in The Virtue of Selfishness, New York, NY: Penguin Group, 1964.

          [10] In my mind, it is a very strong one at that.

          [11] David Friedman, “Some Questions for the Panel,” Cato Unbound, April 10, 2012.

          [12] The intuition behind this being an idea that libertarianism is defined by opposition to the state before anything else. I reject this idea.

          [13] With the caveat from above that if a policy can be shown to be logically required by libertarian principles, then it is a requirement of libertarianism. However, I have not been persuaded by the arguments that anarchism is necessary to meet this requirement.

          More Libertarian Than Thou

          Alexander argues that while it may be possible to describe one view as more libertarian than another, such comparisons hold only at the level of principles, not of policies. In particular, he denies that anarchism is more libertarian than minarchism. One cannot describe one policy as more libertarian than an alternative, he maintains, unless that policy is “logically required by libertarian principles.”

          This position strikes me as unduly restrictive. Suppose two different policies are each minimally consistent with libertarian principles, but one more fully embodies the spirit of those principles than the other does; doesn’t this fact entitle the first to be considered the more libertarian of the two – just as, of two beautiful landscapes, one may be more beautiful than the other; or of two large animals, one may be larger than the other; or of two brilliant theories, one may be more brilliant than the other; or (to cite a eudaimonist example) of two human modes of life, one may be more human than the other?

          Whether anarchism is logically required by libertarian principles is going to depend on what those principles are, a question I’ll let stand for now. But even granting arguendo that anarchism is not thus logically required, I think there are still good reasons to consider anarchism as being at least prima facie more libertarian than minarchism.

          Libertarians surely favour, in general, opening up coercively monopolistic industries to free competition – typically both for consequentialist reasons (competition renders industries more efficient and less prone to abuses) and for natural-rights reasons (whatever one has a right to do, all have a right to do). If theorist A favoured competition in the clothing industry generally, while theorist B favoured competition for most items of clothing but made an exception for shoes, holding that the manufacture and sale of shoes should be a coercive monopoly, then all other things being equal it would be natural to say that theorist A is more libertarian than theorist B.

          Since anarchists want to open up the provision of security (including legal adjudication and rights protection) to competition, while minarchists prefer to leave these services under state monopoly, then all other things being equal it again seems natural to say the anarchists are more libertarian than the minarchists.

          Admittedly such a conclusion is in principle defeasible. If it could be shown that there is something special about security such that free competition here is unworkable, or unstable, or likely to lead to a breakdown of social order, then I do not think it would be obviously right to say that anarchism is more libertarian than minarchism. So the question of which positions are most libertarian cannot always be settled independently from questions about how they would work if implemented. That’s why I applied the term prima facie to anarchism’s status as more thoroughgoingly libertarian.

          But suppose it turns out (as so far it seems to have done) that the arguments for security’s being a special case fail, and that there is no inherent reason to treat the market for security as different from the market for shoes or insurance. In that case, the question of whether to be an anarchist as opposed to a minarchist will simply be one of deciding to open up one more monopolised field to competition. In that case, could one seriously claim that anarchism is no more libertarian than minarchism?

          Clarity on How to Justify Economic Systems

          Alexander writes:

          Based on Z&T’s first article it seems as though there are five potential criteria for moral justification of an economic system other than one based strictly on deontic rights protection:

          1. The system delivers the greatest payout to most well-off in society.

          2. The system delivers the greatest payout to the average members in society

          3. The system delivers the greatest payout to the least well-off in society

          4. The system delivers the greatest payout to everyone in society (i.e. including the least, average, and most well-off) …”

          I do not know what either #2 or #4 is supposed to mean.

          In #2, does “average members” mean “median members”? If so, neither I nor anybody else I have ever heard of advocates that criterion. If, on the other hand, what it means is “the greatest payout, averaged over all members of society,” then it corresponds to one version of utilitarianism (assuming “payout” means “utility”), but his later claim that #2 is consistent with “the moral justification of free market institutions is logically independent from any claims about the effects of those institutions on the material holdings of the poor” is false, since the poor go into the average along with everyone else.

          Does #4 mean “every single person in the society is better under the system than under any other system?” If so, it is unlikely that any system would satisfy it. The only other sense I can make of it is that it means “the greatest payout averaged over everyone in the society,” in which case we are back with utilitarianism and the more reasonable interpretation of #2.

          As far as I can tell, either two of his criteria are identical or one, at least, makes no sense and corresponds to nothing anyone here or elsewhere is arguing for.

          Alexander goes on to write “David Friedman seems to be advocating #2 with his defense of utilitarianism.”

          I am not defending utilitarianism, as ought to be obvious from my earlier post—if I were, I would have to hold that the society (or act or rule) that resulted in more utility was always better. I am arguing that, insofar as something beyond respect for rights ought to be included in the views of libertarians, that something looks more like utilitarianism than like social justice.

          Like Alexander, I too “am interested in asking Z&T to clarify which of these standards they think is most appropriate for evaluating an economic system. Do they want to argue that we must give greater moral weight the least well-off or do they want to argue that we should treat everyone with equal moral weight, which includes the least-well off?” I asked earlier what “social” adds to “justice,” and I do not think I have yet gotten an answer.

          Some Final Thoughts

          by Matt Zwolinski

          We should probably start by thanking Cato Unbound, and thanking Roderick, David, and Alexander for the roles they have played in facilitating this conversation. It has been exciting, exhausting, inspiring, and bewildering. And it has given John and me a tremendous amount of food for thought as we continue to work on our book.

          It is worth re-emphasizing, perhaps, that the book John and I are writing is a historical one. In it, we are not trying to defend a particular conception of social justice as the most philosophically defensible standard, or the most “truly libertarian” one. We are simply trying to trace the ideas as they have developed through the libertarian intellectual tradition. We have our own views, of course, on the substantive philosophical questions. But our goal in our book, and in our essays here, was neither to articulate nor to defend those views.

          Still, like Jason Brennan, we are somewhat puzzled by David’s puzzlement regarding what we mean by the term “social justice.” At the very least, the broad meaning of the concept ought to be clear by now: social justice is a moral standard by which the institutions of a society can be evaluated on the basis of how well they serve the interests of the poor and least advantaged. This broad concept can be fleshed out in a number of different ways by different particular conceptions of social justice. And a full conception would say, among other things, what counts as “advantage” (Wealth? Primary goods? Utility?), what the scope of social justice is (The nation? Humankind? All sentient beings?), how this standard of moral evaluation fits alongside others (nobody – not even Rawls – believes that the fate of the poor is the only important criterion for judging the morality of a society’s institutions), and so on. Again, we have not attempted to articulate or defend such a conception here. But it is not as though bleeding heart libertarians have been silent on this issue. For several serious scholarly treatments, see John’s book here, or this essay by Jason Brennan and John Tomasi. And see also the numerous blog posts from Jason (here, here, and here), Kevin Vallier (here and here) and me (here, here, here and here).

          Our historical thesis is not that earlier classical liberals endorsed any particular conception of social justice. Indeed, we do not even claim that they were explicitly and self-consciously committed to even the broad concept of social justice. But they did, over and over again, suggest that they saw the fate of the working poor as an important element in assessing the justice of liberal institutions. We think that this is an important element of libertarian thought that, at the very least, receded into the background in the twentieth century with figures like Mises, Rand, and Rothbard. And we think that it is beginning to move to the fore again with the neoclassical liberal movement.

          Libertarianism as an Overlapping Consensus

          So when I wrote in my last essay that there is considerable overlap between utilitarians, Rawlsians, and natural rights theorists regarding concern for the poor, I wasn’t trying to dodge David’s questions, as he has suggested elsewhere (here and here). Rather, to borrow some terminology from Alexander, I was trying to make the point that there was a good deal of consensus among the classical liberals regarding the principle that special concern for the poor was warranted, even if there was a great deal of disagreement regarding the proper justification of that principle. A utilitarian might think that we should be especially concerned for the poor because of considerations of diminishing marginal utility, or because the poor are the ones who tend to suffer the most from legal restrictions on the market like . For a Rawlsian, or for a certain type of natural rights theorist, the justification for special concern for the poor will be less contingent some ways. Even a Nozickian or a Rothbardian will feel some attraction to this principle when considering certain cases of historical injustice. Their justifications will differ. But at least for a very broad range of cases, these varying justifications will converge on the same guiding principle: that in assessing the justice of public policy, the needs of the poor and least advantaged have a special priority.

          In carefully constructed hypothetical scenarios of the sort David has been pressing us on, these different justifications will lead their adherents to come to different conclusions regarding the justice of certain institutional structures. And it is a philosophically worthwhile enterprise to trace these differences out, and to try to come to some conclusion regarding which principle, ultimately, we regard as most defensible.

          But libertarians who focus too narrowly on this philosophical project risk losing sight of a tremendously important fact: in the real world, there is a tremendous degree of overlap among the various justificatory approaches, such that libertarians can agree on principles for diagnosing injustice and for making significant progress toward remedying it without settling disagreements about the deep justification of those principles. We agree with Alexander that part of the reason for classical liberalism’s success as a political philosophy is that its principles ring true from a number of different philosophical perspectives. We believe the same is true of social justice, as we have broadly defined it here. Just as individuals with different moral, religious, and metaphysical views can and do converge on principles that support individual liberty, private property, and free markets, so too can and do they converge on a principle that gives special priority to the claims of the poor and disadvantaged in assessing the justice of social institutions. This ability to serve as a point of convergence, we believe, is a sign of and a source of the strength of libertarian theory, and so too of the principle of social justice.

          Absolutism Again

          In an earlier essay, we claimed that 20th century libertarians like Rand, Rothbard, and Mises embraced property rights in an absolutist and monistic way, and that this is part of what distinguishes them from their predecessors in the classical liberal tradition, and also part of what accounts for their rejection of social justice. Roderick has, in his several essays, given us some reason to believe that the difference between our “Unholy Trinity” and the earlier classical liberals is not quite as sharp and clear as we may have made it out to be. And for this we can only thank him. Undoubtedly we did state our case too strongly in our original essay. And refinements of the sort that Roderick’s thoughtful comments have provided us with were exactly what we were hoping to get out of this conversation. We continue to believe that there is an important difference between the postwar libertarians and the earlier classical liberals, and that the characterization we provided goes a good way toward explaining this difference, but simple and clean generalizations are no doubt too much to hope for in doing intellectual history.

          But Roderick also questions the very cogency of the concept of absolutism we employ. Is the concept meant to be a comparative one? How does calling a right absolute differ from specifying the objects to which the right applies? And aren’t all rights absolute anyways, since that is what it means to call something a right?

          Roderick’s first two questions raise important points. Absoluteness does, as we suggested earlier, seem to be something that comes in degrees. To call a right absolute is to say that it trumps other moral considerations. But almost nobody thinks that rights trump all other considerations. My right to free speech means that I get to publish what I want on the internet even if it annoys or offends you. But if for some reason the fate of some thousands of innocent people hinged on censoring me, presumably most of us would think we were morally justified in doing so. So, for most of us, rights are absolute only over a certain range of circumstances or competing situations. And rights can be more or less absolute depending on how expansively that range is specified.

          Now, classical liberals have long (and correctly) noted that there is a point to having property rights that are relatively absolute. Maybe, in theory, bread should belong to whoever is hungriest. As David Hume wrote,

          ’Twere better, no doubt, that every one were possess’d of what is most suitable to him, and proper for his use: But besides, that this relation of fitness may be common to several at once, ’tis liable to so many controversies, and men are so partial and passionate in judging of these controversies, that such a loose and uncertain rule wou’d be absolutely incompatible with the peace of human society. The convention concerning the stability of possession is enter’d into, in order to cut off all occasions of discord and contention; and this end wou’d never be attain’d, were we allow’d to apply this rule differently in every particular case, according to every particular utility, which might be discover’d in such an application.

          David Schmidtz makes essentially the same point in more contemporary language:

          property rights are like traffic lights. Traffic lights move traffic not so much by turning green as by turning red. Without traffic lights, we all in effect have a green light, and the result is gridlock. By contrast, a system where we in turn face red and green lights is a system that keeps us moving. It forces us to stop from time to time, but we all gain in terms of our ability to get where we want to go, because we develop mutual expectations that enable us to get where we want to go, uneventfully. Red lights can frustrate, but the game they create for us is positive-sum. We all get where we are going more quickly, more safely, and more predictably, in virtue of knowing what to expect from each other.

          Traffic lights don’t serve their purpose if people believe that they are entitled to drive through them whenever they’ve got somewhere important to go. But we can all easily think of cases where someone really would be justified in driving through a red light. A certain degree of absoluteness in property rights can thus serve morally important goals. But there is no deep moral or conceptual fact impelling us to be property rights absolutists in the most thoroughgoing sort of way. Rights entail duties, and duties specify very strong reasons for action. But we need not understand them as specifying all-things-considered reasons for action. Sometimes rights will conflict, and we will have to decide which to respect and which to infringe. And sometimes even non-rights-based moral considerations will be weighty enough to warrant violating a right. It would be comforting if we could specify a formula that told us under exactly what circumstances we were justified in doing so. But neither we nor, I suspect, our interlocutors in this dialogue have any such formula that is not prone to counterexample.

          Simple Rules for a Complex World

          Libertarians sometimes worry that by allowing concern for the poor to play an important role in their theory of justice they are opening the floodgate that will inevitably let loose the tide of socialism. And this worry is not without reason. A commitment to social justice does not logically entail a commitment to expansive government action, but public sentiment and public policy are rarely guided by strict rules of logical entailment. A well-meaning and philosophically rigorous conception of social justice can and likely will be transformed into something unrecognizable and indefensible by the politicians, bureaucrats, and lawyers charged with interpreting and enforcing it.

          But this is not a good reason for libertarians to reject the idea of social justice. We do not, after all, believe that socialism really would serve the interests of the poor and disadvantaged better than capitalism. Why, then, should we cede to the socialists the title of the poor’s defenders?

          If our best philosophical, economic, and political reasoning suggests that concern for the poor is best served by placing stringent checks on the power of government to consciously attempt to advance the interests of the poor, then that is the policy required by a commitment to social justice. A moral value that cannot be directly pursued by public policy is nevertheless a moral value. And we do no credit to libertarianism as a philosophical doctrine by denying that it is. There may be good reasons to have our public policies guided by simple rules, but there is no good reason to expect the morality that underlies those rules to be equally simple.

          The greatest minds of the classical liberal tradition— the Smiths, the Humes, the Hayeks—advocated simple rules for the governance of society, but did so while demonstrating remarkable sensitivity to the complexity of our moral and empirical world. In so doing, of course, they faced an enormously difficult challenge. It is no small feat to demonstrate how a severely limited government could possibly be adequately responsive to the unlimited complexities of our moral and economic lives. But there is no shortcut to this challenge. If morality is complex, then libertarian moralists must embrace that complexity and show why their system best accommodates it. And if, as so many thinkers from so many diverse perspectives have concluded, part of our complex set of moral duties includes an obligation to arrange social institutions in a way that serves the interests of the poor and disadvantaged, then libertarians should (and have!) embrace(d) that too. It is a challenge. But it is also an opportunity. And, if we are being philosophically responsible, it is our only option.

            Three Parthian Shots

            My thanks also, to Cato and to my interlocutors. I want to close by emphasizing three points.

            First: although, as Matt and John rightly stress, a conception of justice that is insensitive to issues of human welfare is no morality for human beings, it’s also true that a conception of human welfare that is insensitive to issues of justice is likewise no morality for human beings. That is why I find the approaches of Bentham and of Rawls similarly problematic: they both treat the content of human welfare as something given, something that can be defined independently of moral considerations. In this they indeed follow the dominant trend of post-Renaissance moral philosophy, in both its consequentialist and deontological incarnations: while (rightly) viewing morality as something requiring complex analysis, such approaches treat happiness as something fairly obvious, and in any case something whose nature can be presupposed before turning to moral issues. A richer and more promising picture of value is indicated by the eudaimonist tradition of thinkers like Plato, Aristotle, Cicero, and Aquinas, who saw the concept of happiness as no less complex than, and as inherently entangled with, that of justice. The sensitivity of justice to welfare needs to be a two-way street here, and to the extent that this is recognised, the drift toward excessive consequentialism is averted.

            Second: in its relative inattention to what Nozick called “historical” and I would call “causal” or “agential” issues, the Rawlsian approach neglects the fertile field of class analysis. Once we realise that the inequalities that beset contemporary capitalist society are not merely inefficiencies in the system, but on the contrary are the result of systematic expropriation and exploitation of the less affluent (and less politically connected) by the more so, both the right-wing temptation to dismiss distributional concerns as driven by “envy” and the left-wing temptation to seek to redress such problems via appeal to the very state instrumentalities that produce and perpetuate them will be more effectively forestalled.

            Third: there is a widespread tendency, both on the part of the opponents of bleeding-heart libertarianism and on the part of many of its adherents, to think that the increased sensitivity to the welfare of the poor advocated by BHL naturally leads to a less radical or less “absolutist” version of libertarianism, one more open to exceptions or even to state action. That this is not so is evident from the examples of Thomas Hodgskin, Herbert Spencer, Gustave de Molinari, Benjamin Tucker, and Lysander Spooner in the 19th century; Karl Hess, Sam Konkin, and 1960s-version Rothbard in the 20th; or Kevin Carson, Gary Chartier, Sheldon Richman, and Charles Johnson today. Once the virtually inevitable role of the state as a tool of plutocratic interests is recognised, the effect of social-justice considerations on libertarianism is less to blunt its radicalism than to egg it on.

            So that’s my prescription for libertarianism’s “where next?”: Aristotle, agora, anarchy, action!